SlideShare a Scribd company logo
Indian Polity
For more info: www.iasnext.com Page 1 Ph. :0522 4241011/9454721860
1. The Cabinet Mission to India was headed by :
(a) Stafford Cripps
(b) A.V. Alexander
(c) Lord Pethick Lawrence
(d) Hugh Gaitskell
2. The Constituent Assembly for undivided India
first met on
(a) 6th December, 1946 (b) 9th December, 1946
(c) 20th February, 1947 (d) 3rd June, 1947
3. When the Constituent Assembly for the
Dominion of India reassembled on 31st
October,
1947, its reduced membership was:
(a) 299 (b) 311
(c) 319 (d) 331
4. How long did the Constituent Assembly take to
finally pass the Constitution?
(a) about 6 months in 1949
(b) exactly a year since Nov 26, 1948
(c) about 2 years since Aug 15, 1947
(d) about 3 years since Dec 9, 1946
5. Who presided over the inaugural meeting of the
Constituent Assembly of India?
(a) Sachidananda Sinha (b) B. R. Ambedkar
(c) Dr. Rajendra Prasad (d) P. Upendra
6. The Constitution of India was enacted by a
Constituent Assembly set up :
(a) under the Indian Independence Act, 1947
(b) under the Cabinet Mission Plan, 1946
(c) through a resolution of the provisional
government
(d) by the Indian National Congress
7. Who among the following was the Chairman of
the Drafting Committee of the Indian Constitution?
(a) Rajendra Prasad (b) Tej Bahadur Sapru
(c) C. Rajagopalachari (d) B. R. Ambedkar
8. Match the following:
Constitutional Provisions Sources
A. Federation 1. USA
B. Parliamentary 2. Britain
C. Directive Principle 3. Canada
D. Fundamental Rights 4. Ireland
A B C D
(a) 2 1 4 1
(b) 2 3 1 4
(c) 3 4 2 1
(d) 3 2 4 1
9. Which of the following features and their source
is wrongly matched?
(a) Judicial review – British practice
(b) Concurrent List – Australian Constitution
(c) Directive Principles – Irish Constitution
(d) Fundamental Rights – US Constitution
10. Which Constitution has inspired the Indian
concepts of rule of law, parliamentary system and
law-making procedure?
(a) US Constitution (b) British Constitution
(c) Canadian Constitution (d) All of the above
11. Which of the following emerges clearly from
the Preamble?
1. When the Constitution was enacted.
2. The ideals that were to be achieved.
3. The system of government
4. The source of authority
(a) II, III and IV (b) I and II
(c) I, II and III (d) I, II, III and IV
12. Which one of the following statements
regarding the Preamble is correct?
(a) It is not enforceable in a court of law
(b) the Supreme Court has recently ruled that it is
not a part of the Constitution
(c) It has been amended twice
(d) All the above are correct
13. Match the following:
Constitutional Features Sources
A. Rule of Law 1. German Constitution
B. Directive Principles 2. Australian Constitution
C. Concurrent List 3. Irish Constitution
D. Suspension of 4. British Constitution
Fundamental Rights
during Emergency
A B C D
(a) 4 2 1 3
(b) 4 3 2 1
(c) 2 3 4 1
(d) 2 1 4 3
14. Which of the following features does the Indian
Constitution borrow from the Weimar Constitution
of Germany?
(a) The idea of a federation with a strong centre
(b) The method of presidential elections
(c) The Emergency powers
(d) Provisions concerning the suspension of
fundamental rights during National Emergency
15. The Constitution of India is designed to work as
a unitary government:
(a) in normal times
(b) in times of Emergency
(c) when the Parliament so desires
(d) at the will of the President
16. Which of the following features of the
Constitution of India does not resemble the
American Constitution?
(a) Written Constitution
(b) Federal form of government
(c) Fundamental Rights
(d) Parliamentary form of government
17. Which of the following statements regarding
the acquisition of Indian citizenship is correct ?
(a) If citizenship is to be acquired by registration,
six months residence in India is essential.
Indian Polity
For more info: www.iasnext.com Page 2 Ph. :0522 4241011/9454721860
(b) If citizenship is to be acquired by naturalisation
the person concerned must have resided in India for
5 years.
(c) If born in India, only that person can acquire
citizenship at least one of whose parents is an
Indian.
(d) Rules regarding the citizenship of India shall
not be applicable to Jammu and Kashmir.
18. According to the Citizenship Act, 1955, Indian
citizenship cannot be acquired by:
(a) children of the diplomatic personnel or alien
enemies born in India on or after 26th January,
1950
(b) children born of Indian citizens abroad
(c) incorporation of territory not forming part of
India at the commencement of the Constitution
(d) naturalization
19. Details on how citizenship may be acquired and
terminated are enumerated in:
(a) Part II of the Constitution
(b) the Citizenship Act, 1955
(c) Schedule I of the Constitution
(d) various enactments by Parliament
20. Which of the following is/are included in the
Directive Principles of State Policy? [IAS 2008]
1. Prohibition of traffic in human beings and forced
labour
2. Prohibition of consumption except for medicinal
purposes of intoxicating drinks and of other drugs
which are injurious to health.
(a) 1 only (b) 2 only
(c) Both 1 and 2 (d) Neither 1 nor 2
21. The President of the Indian Union has the same
constitutional authority as the:
(a) British Monarch (b) President of USA
(c) President of Egypt (d) President of Russia
22. Article 6 enshrines the provision conferring
Rights of citizenship of certain persons who have
migrated to India from Pakistan. Which one of the
following provisions is incorrect?
(a) He or either of his parents or any of his grand-
parents was born in India as defined in the
Government of India Act, 1935
(b) Such person has so migrated before 19 July,
1948
(c) Such person has so migrated on or after the 19
July, 1948, he has been registered as a citizen
(d) Such person has got married in India
23. Which Schedule of the Constitution of India
contains special provisions for the administration
and control of Scheduled Areas in several States?
(a) Third (b) Fifth
(c) Seventh (d) Ninth
24. Indian citizenship of a person can be terminated
if:
1. a person voluntarily acquires the citizenship of
some other country.
2. a person who has become a citizen through
registration is sentenced to imprisonment for not
less than 10 years within five years of his
registration.
3. the Government of India is satisfied that
citizenship was obtained by fraud.
4. a person who is a citizen by birth indulges in
trade with an enemy country during war.
(a) I and III (b) I, II and III
(c) I, III and IV (d) I, II, III and IV
25. Citizenship provisions are enshrined in the
Constitution in :
(a) Part II (b) Part II, Articles 5-11
(c) Part II, Articles 5-6 (d) Part I, Articles 5-11
26. As far as Armed Forces are concerned, the
fundamental rights• granted under Articles 14 and
19 of the Constitution are:
(a) not available at all
(b) available to armed forces but not to other forces
(c) available only at the discretion of the chief of
army staff
(d) available only according to law made by
Parliament
27. Which of the following Articles describes about
the person voluntarily acquiring citizenship of a
foreign state not to be citizens?
(a) Article 5 (b) Article 7
(c) Article 8 (d) Article 9
28. Which of the following statements is/are not
indicative of the difference between Fundamental
Rights and Directive Principles?
I. Directive Principles are aimed at promoting
social welfare, while Fundamental Rights are for
protecting individuals from State encroachment.
II. Fundamental Rights are limitations on State
action, while Directive Principles are positive
instructions for the Government to work towards a
just socioeconomic order.
III. Fundamental Rights were included in the
original Constitution, but Directive Principles were
added by the first Amendment.
IV. Fundamental Rights are amendable, but
Directive Principles cannot be amended.
(a) I and II (b) II and III
(c) III and IV (d) I, II and III
29. Consider the following statements in respect of
financial emergency under Article 360 of the
Constitution of India:
1. A proclamation of financial emergency issued
shall cease to operate at the expiration of two
months, unless before the expiration of that period
it has been approved by the resolutions of both
Houses of Parliament.
2. If any proclamation of financial emergency Is in
operation. it is competent for the President of India
to issue directions for the reduction of salaries and
allowances of all or any class of persons serving in
connection with the affairs of the Union but
Indian Polity
For more info: www.iasnext.com Page 3 Ph. :0522 4241011/9454721860
excluding the Judges of the Supreme Court and the
High Courts.
Which of the statements given above is/are correct?
(a) 1 only (b) 2 only
(c) Both 1 and 2 (d) Neither 1 nor 2
30. Part V of the Constitution deals with:
I. Union Executive
II. Parliament
III. Supreme Court and High Courts
IV. Comptroller and Auditor-General
(a) I and II (b) I, II and III
(c) I only (d) I, II and IV
31. Match the following -
Articles of the Constitution of India
1. Article 14
2. Article 15
3. Article 16
4. Article 17
Provision
1. The State shall not discriminate against any
citizen on grounds only of religion, race, caste, sex,
place of birth or any of them
2. The State shall not deny to any person equality
before the law or the equal protection of laws
within the territory of India
3. 'Untouchability' is abolished and its practice in
any form is forbidden
33. There shall be equality of opportunity for all
citizens in matters relating to employment or
appointment to any office under the State
A B C D
(a) 2 4 1 3
(b) 3 1 4 2
(c) 2 1 4 3
(d) 2 4 1 2
32. The Governor recommends to the President of
India that breakdown of Constitutional machinery
in the State is imminent. The President makes a
proclamation under Article 356. The action of the
President:
(a) cannot be reviewed as the President is the sole
judge of his emergency powers
(b) cannot be reviewed as the Constitution bars the
courts from reviewing political actions
(c) can be reviewed as it goes against Fundamental
Rights
(d) can be reviewed as it is malafide
33. Provisions of having a UPSC and Public
Service Commision for States are enshrined in :
(a) Part XIV, Chapter II, Articles 315-324
(b) Part XIV, Chapter I, Articles 308-323
(c) Part XIV, Chapter II, Articles 315-323
(d) Part XIV, Chapter I, Articles 308-318
34. Match the following:
A. Part I 1. Fundamental Rights
B. Part III 2. Panchayati Raj
C. Part IX 3. Citizenship
D. Part II 4. The Union and its Territory
A B C D
(a) 4 2 3 1
(b) 3 1 2 4
(c) 2 3 1 3
(d) 4 1 2 3
35. Match the following:
A. Article 61 1. Removal of Deputy Chairman of
Rajya Sabha
B. Article 67 2. Impeachement of President
C. Article 94 3. Removal of Vice President
D. Article 90 4. Removal of Speaker
A B C D
(a) 2 3 4 1
(b) 1 3 4 1
(c) 2 1 4 3
(d) 3 4 2 1
36. Match the following:
A. Abolition of Untouchability 1. Article 24
B. Abolition of Titles 2. Article 23
C. Prohibition of Child labour 3. Article 17
D. Prohibition of Traffic in 4. Article 18
human beings
A B C D
(a) 3 4 2 1
(b) 2 4 1 3
(c) 3 4 1 2
(d) 1 3 2 4
37. During financial emergency, the President can:
1. ask the states to reduce the salaries arid
allowances of all or any class of persons serving in
connection with the affairs of the State
2. ask the states to reserve money bills passed by
the state legislature for his consideration
3. issue directions to states on financial matters
4. issue directions for the reduction of salaries and
allowances of persons serving in connection with
the affairs of the Union
(a) I, II and III (b) I, III and IV
(c) II, III and IV (d) I, II, III and IV
38. During a proclamation of emergency due to the
breakdown of constitutional machinery in a State
the President can
I. assume all powers vested in and exercisable by
the Governor
II. declare that the powers of the State Legislature
shall be exercised by Parliament
III. assume certain powers of the High Courts
IV. suspend by order any or all Fundamental Rights
except those under Articles 20 and 21
(a) I and II (b) I and III
(c) II, III and IV (d) II and IV
39. The value of a vote of a Member of Parliament
for the election of the President is determined by
dividing the:
(a) nation's population as per the latest census by
the number of Lok Sabha members
(b) nation's population as per the latest census by
the total strength of the two Houses of Paliament
Indian Polity
For more info: www.iasnext.com Page 4 Ph. :0522 4241011/9454721860
(c) the total value of votes of members of all the
State Legislative Assemblies by the elected
Members of the two Houses of Parliament
(d) particular State's population as per the latest
census by the number of Members of Parliament
elected from that State
40. Who among the following' are appointed by the
President of India?
I. Chairman, Finance Commission
II. Deputy Chairman, Planning Commission
III. Chief Minister of a Union Territory
(a) I only (b) I and II
(c) I and III (d) II and III
41. Which of the following is/are resultant of a
proclamation of National Emergency because of
war?
I. The Union Government can give directions to the
States about how the executive power of theState is
to be exercised.
II. The Fundamental Rights stand automatically
suspended.
III. The State Legislature is suspended.
IV. Parliament can make laws with respect to any
subject in the State List.
(a) I, II and IV (b) I, III and IV
(c) I only (d) I and IV
42. In case of proclamation of emergency on
grounds of war or external aggression:
(a) all Fundamental Rights will be automatically
suspended
(b) the right to move a court for enforcement of any
Fundamental Right is suspended
(c) the President may order the suspension of
enforcement of any Fundamental Right except
Article 20-21
(d) Parliament may authorise suspension of all
Fundamental Rights
43. The President of India is:
(a) Commander-in-Chief of Defence Forces
(b) Supreme Commander of Armed Forces
(c) Head of the Government
(d) Supreme Commander of Defence Forces of the
Union and Executive Head of the Union
44. Which of the following statements regarding
the pardoning powers of the President is/are not
correct?
1. He has the pardoning power in respect of
sentence by court martial.
2. He can grant reprieve and respite in case of
punishment for an offence against any law of the
land, Union or State.
3. He alone can pardon a sentence of death.
4. His exercise of the power of pardon is open to
judicial review.
(a) I and III (b) II and III
(c) I and IV (d) II and IV
45. The President of India is vested with ordinance
making power by Article 126, He exercises this
power:
(a) when Lok Sabha is not in session and
circumstances .exist which render it necessary for
the President to take immediate action
(b) when Council of States is not in session and the
President is satisfied that circumstances exist which
render it necessary for him to take immediate
action
(c) when both Houses of Parliament are not in
session and President is satisfied that circumstances
exist which made it necessary for him to take the
immediate action
(d) in all the above circumstances
46. Consider the following statements about the
powers of the President of India:
1. The President can direct that any matter on
which decision has been taken by a Minister should
be placed before the Council of Ministers.
2. The President can call all information relating to
proposals for legislation.
3. The President has the right to address and send
messages to either House of the Parliament.
4. All decisions of the Council of Ministers relating
to the administration fo the Union must be
communicated to the President.
Which of the statements given above are correct?
(a) 1, 2 and 3 only (b) 1 and 3 only
(c) 2 and 4 only (d) 1, 2, 3 and 4
47. The Prime Minister, at the time of the
appointment:
1. need not necessarily be a member of one of the
Houses of Parliament but must become a member
of one of the Houses within six months.
2. need not necessarily be a member of one of the.
Houses of Parliament but must become a member
of the Lok Sabha within six months.
3. must be either a nominated or elected member of
one of the Houses of Parliament.
4. must be an elected member of only Lok Sabha.
(a) I only (b) I and III
(c) II only (d) IV only
48. Consider the following Vice-President of India:
1. V.V. Giri 2. M. Hidayatullah
3. B.D. Jatti 4. G.S. Pathak
Which one of the following is the correct
chronology of their tenures?
(a) 1-4-3-2 (b) 2-1-3-4
(c) 3-2-1-4 (d) 4-1-3-2
49. Who among the following have held the office
of the Vice-President of India?
1. Mohammad Hidayatullah
2. Fakhruddin Ali Ahmed
3. Neelam Sanjiva Reddy
4. Shankar Dayal Sharma
(a) 1, 2, 3 and 4 (b) 1 and 4
(c) 2 and 3 (d) 3 and 4
Indian Polity
For more info: www.iasnext.com Page 5 Ph. :0522 4241011/9454721860
50. Which of the following committees exist only
in the Lok Sabha?
1. Business Advisory Committee.
2. Committee on Private Members Bill and
Resolutions.
3. Committee on Petitions.
4. Committee on Assurances.
5. Estimates Committee.
(a) I, II and III (b) III and IV
(c) II and V (d) II, IV and V
51. Each member of Parliament who participates in
the Presidential election is entitled to cast as many
votes as are obtained by dividing the total number
of votes of the Legislative Assemblies of all the
States by the total number of elected members of
the two Houses of Parliament. This ensures:
(a) parity between the voting strengths of the States
and the Parliament
(b) parity among the States
(c) uniformity of representation of the different
States
(d) All the above
52. What is the minimum percentage of seats a
party should get to be recognised as the opposition
party in the legislature?
(a) 20% (b) 15%
(c) 10% (d) No such limit
53. The maximum number of representatives of the
States in Lok Sabha is :
(a) 525 (b) 530
(c) 545 (d) 550
54. Match the following:
Parliamentary Terms
A. Starred Question
B. Unstarred Question
C. Short Notice Question
Meaning
1. One asked by a member on matters of public
importance of an urgent nature
2. One for which the Concerned minister has to lay
on the table a written answer
3. One for which an oral answer is required to be
given by a minister on the floor of the House
A B C
(a) 1 2 3
(b) 2 1 3
(c) 3 1 2
(d) 3 2 1
55. Who among the following was never the Lok
Sabha Speaker?
(a) K.V.K. Sundaram (b) G.S. Dhillon
(c) Balirarn Bhagat (d) Hukarn Singh
56. The Union Territories get representation in:
(a) Lok Sabha
(b) Rajya Sabha
(c) both Houses of Parliament
(d) None of the above
57. Which of the following statements regarding
the Esstimates Committee are correct?
1. No member of the Rajya Sabha is associated
with it.
2. It has twenty members.
3. Its members are elected in accordance with the
system of proportional representation for a period
of one year.
4. The Speaker nominates one of its members to be
its Chairman.
(a) I, II and III (b) I, III and IV
(c) I, II and IV (d) II, III and IV
58. Match the following:
A. Business Advisory Committee
1. Looks into the mode of public expenditure
B. Select Committee
2. Examines the veracity of budget estimates
C. Estimates Committee
3. Considers a bill and submits to the House a
report on it
D. Public Accounts Committee
4. Prepares time table for the whole session
A B C D
(a) 1 2 3 4
(b) 2 3 4 1
(c) 3 1 2 4
(d) 4 3 2 1
59. Which of the following is the correct sequence
of the grades of officers in the Central Secretariat?
1. Secretary 2. Additional Secretary
3. Joint Secretary 4. Deputy Secretary
5. Under Secretary 6. Director
(a) 1, 2, 3, 6, 4, 5 (b) 2, 1, 3, 5, 4, 6
(c) 6, 4, 3, 5, 2, 1 (d) 1, 5, 4, 2, 6, 3
60. An Appropriation Bill:
I. is necessary to draw money from the
Consolidated Fund of India.
II. cannot be amended to vary the amount of any
charged expenditure.
III. Includes only the expenditure charged on the
Consolidated Fund of India.
IV. is required to withdraw money from the
Contingency Fund of India.
(a) I and III (b) I, II and III
(c) I and II (d) I, II, III and IV
61. With reference to Indian Public Finance,
consider the following statements:
1. Disbursements from Public Accounts of India
are subject to the Vote of Parliament.
2. The Indian Constitution provides for the
establishment of a Consolidated Fund, a Public
Account and a Contingency Fund for each State.
3. Appropriations and disbursements under the
Railway Budget are subject to the same form of
parliamentary control as other appropriations and
disbursements.
Which of the statements given above are correct?
(a) 1 and 2 (b) 2 and 3
(c) 1 and 3 (d) 1, 2 and 3
Indian Polity
For more info: www.iasnext.com Page 6 Ph. :0522 4241011/9454721860
62. Which of the following matters are not affected
in case of dissolution of the Lok Sabha?
1. A bill originating and pending in the Rajya
Sabha.
2. Pending notices, motions and resolutions in the
Lok Sabha.
3. A joint sitting of Parliament if notification of
such a sitting has been given before the dissolution.
4. Bills passed by both Houses and sent to the
President for his assent.
5. Bills returned by President for reconsideration.
(a) I, II and III (b) I, III, IV and V
(c) II, III and IV (d) I, IV and V
63. Which one of the following statements is not
correct?
(a) In Lok Sabha, a no-confidence motion has to set
out the grounds on which it is based
(b) In the case of a no-confidence motion in Lok
Sabha no conditions of admissibility have been laid
down in the Rules
(c) A motion of no-confidence, once admitted, has
to be taken up within ten days of the leave being
granted
(d) Rajya Sabha is not empowered to entertain a
motion of no-confidence
64. The two Houses of Parliament enjoy co-equal
power in all spheres except:
1. financial matters
2. responsibility of the Council of Ministers
3. amendment procedure
4. election of President
(a) III and IV (b) II, III and IV
(c) I, II and III (d) I and II
65. What is the minimum age for being the member
of the Parliament?
(a) 21 years (b) 25 years
(c) 30 years (d) 35 years
66. The total number of members in a Legislative
Council should not exceed that of a Legislative
Assembly by :
(a) 1/3 (b) 2/3
(c) 1/2 (d) 1/4
67. Which one of the following is the largest (area
wise) Lok Sabha constituency?
(a) Kangra (b) Kachchh
(c) Ladakh (d) Bhilwara
68. Match the following:
Jurisdiction of the Supreme Court
A. Original Jurisdiction
B. Appellate Jurisdiction
C. Advisory Jurisdiction
Cases Covered
1. Advice on any question of law as may be
referred to the Supreme Court for consideration by
the President
2. Case involving interpretation of the Constitution
3. Appointment of officers and servants of the
Supreme Court
4. Dispute between the Government of India and a
State
A B C
(a) 1 2 3
(b) 4 3 2
(c) 4 2 1
(d) 2 1 3
69. Which of the following correctly reflects the
position of the Upper House of the State
Legislature as compared to the position of the
Upper House of the Parliament?
(a) 1/3rd members of Rajya Sabha retire every
second year while 1/4th members of Legislative
Council retire every 18 months
(b) There is no provision for a joint sitting of the
Legislative Assembly and the Legislative Council
in the States for resolving deadlocks While there is
such a provision in the case of the two House of
Parliament
(c) While no Bill other than a Money Bill can
originate in the State Legislative Council, no Bill
can originate in the Rajya Sabha
(d) While Rajya Sabha has 12 nominated members,
the State Legislative Council has none
70. If in an election to a State Legislative Assembly
the candidate who is declared elected loses his
deposit, it means that:
(a) the polling was very poor
(b) the election was for a multi- member
constituency
(c) the elected candidate's victory over his nearest
rival was very marginal
(d) a very large number of candidates contested the
election
71. Match the following:
A. Mandamus 1. Direction to an official for
the performance of a duty
B. Habeas Corpus 2. Release of an illegally
detained person
C. Certiorari 3. Transferring of a case from
an inferior court to a court
of higherjurisdiction
D. Quo Warranto 4. Calling upon one to show
by what authority he
holds or claims
afranchise or office
A B C D
(a) 1 2 3 4
(b) 3 2 1 4
(c) 4 1 3 2
(d) 4 3 2 1
72. Consider the following statements:
1. Justice V. R. Krishna Iyer was the Chief Justice
of India
2. Justice V. R. Krishna Iyer is considered as one of
the progenitors of public interest litigation(PIL) in
the Indian judicial system.
Which of the statements given above is/are correct?
(a) 1 only (b) 2 only
(c) Both 1 and 2 (d) Neither 1 nor 2
Indian Polity
For more info: www.iasnext.com Page 7 Ph. :0522 4241011/9454721860
73. A: The position of the legislative Council is
inferior to that of the legislative Assembly.
R1: The very existence of the Council depends on
the will of the Assembly.
R2: A Bill originating in the Council can be
forthwith put to an end by the Assembly rejecting
it.
R3: One-sixth of the Council's members are
nominated by the Governor.
(a) A and R1, R2 and R3 are correct and R1, R2
and R3 explain A
(b) A, R1 and R3 are correct and R1 and R3
explain A
(c) A, R1, R2and R3are correct but only R1 and R2
explain A
(d) A and R2 are wrong; R1 and R3 are correct
74. The High Courts in India were first started at:
(a) Bombay, Madras, Calcutta
(b) Delhi and Calcutta
(c) Bombay, Delhi, Madras
(d) Madras and Bombay
75. The number of seats in Vidhan Sabha is :
(a) to be not more than five hundred and not less
than 60
(b) to be not more than 500 and not less than 60 but
an exception is recognished in the case of one State
which has only 32 seats
(c) to be not more than.600 and not less than 500
(d) varies from Vidhan Sabha to Vidhan Sabha
76. Who of the following shall cause every
recommendation made by the Finance
Commission to be laid before each House of
Parliament?
(a) The President of India
(b) The Speaker of Lok Sabha
(c) The Prime Minister of India
(d) The Union Finance Minister
77. In which respect have the Centre-State
relations been specifically termed as 'municipal
relations'?
(a) Centre's control of the State in the legislative
sphere
(b) Centre's control of the State in financial matters
(c) Centre's control of the State in administrative
sector
(d) Centre's control of the State in planning process
78. Which of the following is/are extra
constitutional and extra-legal device (s)for securing
co-operation and co-ordination between the States
in India?
1. The National Development Council.
2. The Governor's Conference.
3. Zonal Councils.
4. The Inter-State Council
(a) I and II (b) I, III and IV
(c) III and IV (d) Only IV
79. Match the following
Year of Creation States
A. 1960 1. Sikkim
B. 1962 2. Goa
C. 1975 3. Maharashtra
D. 1987 4. Nagaland
A B C D
(a) 2 4 3 1
(b) 4 3 2 1
(c) 3 4 1 2
(d) 3 4 2 1
80. Match the following:
A. Arunachal Pradesh 1. 22nd State
B. Goa 2. 23rd State
C. Mizoram 3. 24th State
D. Sikkim 4. 25th State
A B C D
(a) 1 3 4 2
(b) 4 3 1 2
(c) 2 1 4 3
(d) 3 4 2 1
81. Corporation Tax:
(a) is levied by the Union and collected and
appropriated by the States
(b) is levied by the Union and belongs to it
exclusively
(c) is levied and appropriated by the States
(d) is levied by the Union and shared by the Union
and the States
82. The States Reorganisation Act created ______
States and _______ Union Territories.
(a) 14;7 (b) 14;6
(c) 15;6 (d) 15;7
83. Silvassa is the capital of
(a) Lakshadweep
(b) Tripura
(c) Dadra and Nagar Haveli
(d) Mizoram
84. What is the correct sequence of steps in
electoral procedure?
1. Filing of nominations.
2. Presidential notification.
3. Scrutiny of nomination papers.
4. Withdrawal of candidature.
5. Allotment of symbols.
(a) I, III, IV, V, II (b) II, I, IV, V, III
(c) I, III, V, IV, II (d) II, I, III, IV, V
85. Match the following:
A. Ex-Officio Chairman of Rajya Sabha
1. Speaker
B. Presiding officer of the House of the People
2. Solicitor General
C. First law officer of the State
3. Vice-President
D. Representative of Government of India in
important legal cases 4. Advocate General
A B C D
(a) 3 1 4 2
(b) 2 1 4 3
(c) 1 3 4 2
Indian Polity
For more info: www.iasnext.com Page 8 Ph. :0522 4241011/9454721860
(d) 4 1 3 2
86. What was the decision of the Supreme Court in
Keshavanand Bharati case?
(a) Parliament is supreme in the matters of
legislation
(b) In matters relating to compulsory acquisition of
private property the court had the ultimate powers
of determining what is public purpose under Article
31
(c) The power under Article 368 to amend the
Constitution cannot be so exercised as to alter the
basic structure or the essential features of the
Constitution
(d) The Supreme Court has full authority to
pronounce on the Constitutional Validity of any
State law
87. Which of the following are a source of income
to the urban local bodies?
1. Octroi duty on goods brought into and taken out
of the city.
2. Taxes on vehicles.
3. Income from water and electricity supply.
4. Financial grants from the State Governments.
(a) I, II and III (b) II, III and IV
(c) I, III and IV (d) I, II, III and IV
88. Match the following:
A. 58th
Amendment 1. Delhi to be called as
National Capital
Territory of Delhi
B. 61st
Amendment 2. Reduced voting age
from 21 years
to 18 years
C. 69th
Amendment 3. An authoritative text
of the Constitution
in Hindi
D. 71st
Amendment 4. Included Konkani,
Manipuri and Nepali
languages in the
Eighth Schedule
A B C D
(a) 3 2 1 4
(b) 3 1 2 4
(c) 2 1 3 4
(d) 4 2 1 3
89. Which of the two words among the following
were added to the Preamble to the Constitution of
India by the Constitution (42nd
Amendment) Act,
1976?
(a) Sovereign and Socialist
(b) Socialist and Democratic
(c) Socialist and Secular
(d) Secular and Democratic
90. The President of India appoints the Chief
Election Commissioner, but he cannot be removed
from office except by a special procedure laid
down in the Constitution. This procedure is the
same as the one prescribed for the removal of the
(a) Vice-President
(b) Judges of the Supreme Court
(c) Members of the State Public Service
Commissions
(d) Members and Chairman of the UPSC
91. The theory of "basic structure" of the
Constitution was propounded by the Supreme
Court in the
(a) A.K.Gopalan Case
(b) Golaknath Case
(c) Kesavananda Bharati Case
(d) Minerva Mills Case
92. The Comptroller and Auditor General (CAG)
does not audit the receipts and expenditure of
(a) Municipal Undertakings
(b) State Governments
(c) Government Companies
(d) Union Government
93. The Constituent Assembly of India was
converted into the Provisional Parliament of India
on
(a) 1.1.1950 (b) 15.8.1947
(c) 2.10.1950 (d) 26.1.1950
94 .The Union or India consists of _____ States and
Union Territories.
(a) 22 : 8 (b) 24 : 7
(c) 28 : 7 (d) 21 : 8
95. During the period of 15th August 1947 to the
26th January 1950, the political status of India was
that of a
(a) Sovereign Republic
(b) Dominion in the British Commonwealth of
Nations
(c) Sovereign Republic and a member of the
Commonwealth
(d) Sovereign State
96. Consider the following tasks:
1) Superintendence, direction and conduct of free
and fair elections.
2) Preparation of electoral rolls for all elections to
the Parliament, State-Legislatures and the Office of
the President and the Vice-President.
3) Giving recognition to political parties, allotting
election symbols to political parties and individuals
contesting the election.
4) Proclamation of final verdict in the case of
election disputes.
Which of the above are the functions of the
Election Commission of India?
(a) 1, 2 and 3 (b) 2, 3 and 4
(c) 1 and 3 (d) 1, 2 and 4
97. In the Government of India, under which
Ministry is the National River Conservation
Directorate?
(a) Ministry of Agriculture
(b) Ministry of Earth Sciences
(c) Ministry of Water Resources
(d) Ministry of Environment and Forests
Indian Polity
For more info: www.iasnext.com Page 9 Ph. :0522 4241011/9454721860
98. The seven Union Territories occupy roughly
______ per cent of the total area of the country.
(a) 12 (b) 0.33
(c) 6 (d) 3
Ans. b
99. The first Independence Day (August 15, 1947)
fell on a
(a) Wednesday (b) Thursday
(c) Friday (d) Saturday
100. Match the following:
Prominent Functionaries Term of Office
A. Supreme Court Judge 1. Till 65 years of age
B. High Court Judge 2. Till 62 years of age
C. Comptroller and 3. 6 years or age of 65
Auditor General of India whichever is less
D. Governor 4. 5 years
A B C D
(a) 1 2 3 4
(b) 1 3 2 4
(c) 2 1 3 4
(d) 3 2 4 1
101. Match the following:
A. Madhu Limayi 1. A former Vice-President
B. Pattabhi Sitaramaiah 2. The first Speaker of the
Lok Sabha
C. B.D. Jatti 3. A veteran parliamentarian
of yester years
D. Mavlankar 4. Author of a famous book
on the history of Indian
National Congress
A B C D
(a) 2 4 3 1
(b) 3 1 2 4
(c) 4 3 1 2
(d) 3 4 1 2
102. In India, if a religious sect/community is given
the status of a national minority, what special
advantages is it entitled to ?
1. It can establish and administer exclusive
educational institutions.
2. The President of India automatically nominates a
representative of the community to Lok Sabha.
3. It can derive benefits from the Prime Minister's
15-Point Programme.
Which of the statements given above is/are correct?
(a) 1 only (b) 2 and 3 only
(c) 1 and 3 only (d) 1, 2 and 3
103. Which of the following condition/conditions
must be fulfilled by the NRIs to be eligible to vote
in elections in India?
1. They must be physically present in their place of
origin to exercise their franchise.
2. NRIs whether they have acquired citizenship of
other countries or not are eligible to vote
3. Eligible NRIs have to register by filling up form
6-A with electoral registration office.
Select the correct answer using the code given
below:
(a) 1, 2 and 3 (b) 1 and 3 only
(c) 2 only (d) 3 only
104. Which one of the following States has granted
Sanskrit language the status of the second official
language of the State?
(a) Bihar (b) Chhattisgarh
(c) Uttar Pradesh (d) Uttarakhand
105. Which of the following Constitution
Amendment Acts seeks that the size of the
Councils of Ministers at the Centre and in a State
must not exceed 15 percent of the total number of
members in the Lok Sabha and the total number of
members of the Legislative Assembly of that State,
respectively?
(a) 91st
(b) 93rd
(c) 95th
(d) 97th
106. Which one of the following is the correct
sequence in the descending order of precedence in
the warrant of precedence?
(a) Attorney General of India - Judges of the
Supreme Court - Members of Parliament Deputy
Chairman of Rajya Sabha
(b) Judges of the Supreme Court - Deputy
Chairman of Rajya Sabha - Attorney General of
India - Members of Parliament
(c) Attorney General of' India - Deputy Chairman
of Rajya Sabha - Judges of the Supreme Court -
Members of Parliament
(d) Judges of the Supreme Court - Attorney
General of India - Deputy Chairman of Rajya
Sabha - Members of Parliament
107. The Constitution (Seventy-Third Amendment)
Act, 1992, which aims at promoting the Panchayati
Raj Institutions in the country, provides for which
of the following?
1. Constitution of District Planning Committees.
2. State Election Commissions to conduct all
panchayat elections.
3. Establishment of State Finance Commissions.
Select the correct answer using the codes given
below:
(a) 1 only (b) 1 and 2 only
(c) 2 and 3 only (d) 1, 2 and 3
108. For which one of the following reforms was a
Commission set up under the Chairmanship of
Veerappa Moily by the Government of India?
(a) Police Reforms
(b) Tax Reforms
(c) Reforms in Technical Education
(d) Administrative Reforms
109. Which of the following is incorrect in respect
of Local Government in India?
(a) According to the Indian Constitution local
government is not an independent tier in the federal
system
(b) 30% of the seats in local bodies are reserved for
women
(c) Local government finances are to be provided
by a Commission
(d) Elections to local bodies are to be determined
by a Commission
Indian Polity
For more info: www.iasnext.com Page
10
Ph. :0522 4241011/9454721860
110. The expression 'Creamy layer' used in the
judgement of the Supreme Court relating to the
case regarding reservations refers to:
(a) those sections of the society which pay income
tax
(b) those sections of socially and educationally
backward classes of the society that are developed
(c) those sections of the society that are considered
advanced according to the Karpuri Thakur formula
(d) all sections of the upper castes of the society
111. The composition of the UPSC is:
(a) laid down in the Constitution
(b) determined by Parliament
(c) determined by the President
(d) determined by the Chairman of the UPSC
112. Which of the following is correct regarding
booth capturing?
1. It has been defined in the Constitution after the
61st amendment.
2. It includes the seizure of a polling booth to
prevent the orderly conduct of elections.
3. It is also committed when any elector is
threatened and prevented from going to the polling
station to cast his vote.
4. It has been declared a cognizable offence
punishable by imprisonment.
(a) 2, 3, and 4 (b) 1, 2 and 3
(c) 2 and 3 (d) 1, 2, 3 and 4
113. The first Law Officer of the Government of
India is:
(a) Chief Justice of India (b) Law Minister
(c) Attorney General (d) Auditor General
114. The declaration which outlines the future
programme and policy of a political party issued on
the eve of a general election is called:
(a) white paper (b) manifesto
(c) yellow paper (d) mandate
115. The old name of which State/Union Territory
is wrongly given?
(a) Karnataka - Mysore
(b) Tamil Nadu - Madras
(c) Lakshadweep - Laccadive, Minicoy and
Amindiv Islands
(d) Meghalaya - Eastern Hill Province
116. Who of the following shall cause every
recommendation made by the Finance Commission
to be laid before each House of Parliament?
(a) The President of India
(b) The Speaker of Lok Sabha
(c) The Prime Minister of India
(d) The Union Finance Minister
117. Which of the following features is/are contrary
to the norms of a federal polity?
1. Common All India Service
2. Single integrated judiciary
Select the correct answer using the code given
below:
(a) 1 only (b) 2 only
(c) Both 1 and 2 (d) Neither 1 nor 2
118. Which one of the following was an associate
State of India before becoming a full fledged State?
(a) Meghalaya (b) Mizoram
(c) Manipur (d) Sikkim
119. The distribution of power between Centre and
the States is based on the scheme provided in :
(a) Government of India Act, 1935
(b) Montague-Chelmsford Act, 1919
(c) Minto-Morley Reforms, 1909
(d) Indian Independence Act, 1947
120. The provisions regarding division of taxes
between Union and the States:
(a) can be suspended during National Emergency
(b) can be suspended during Financial Emergency
(c) can be suspended only with the consent of the
majority of State legislatures
(d) cannot be suspended under' any circumstances

More Related Content

What's hot

Salient features of constitution of India
Salient features of constitution of IndiaSalient features of constitution of India
Salient features of constitution of India
sameer313
 
Lecture 5 citizenship
Lecture 5   citizenshipLecture 5   citizenship
Lecture 5 citizenship
amanbishla1
 
Constitution of india a living document
Constitution of india  a living documentConstitution of india  a living document
Constitution of india a living document
VaasuGupta5
 
Salient features of indian constitutions.
Salient features of indian constitutions.Salient features of indian constitutions.
Salient features of indian constitutions.
Abrar Khan
 
Lecture 2 introduction to indian constitution
Lecture 2   introduction to indian constitutionLecture 2   introduction to indian constitution
Lecture 2 introduction to indian constitution
amanbishla1
 
Lecture 1-historical-background
Lecture 1-historical-backgroundLecture 1-historical-background
Lecture 1-historical-background
amanbishla1
 
Lecture 4 union and its territory
Lecture 4   union and its territoryLecture 4   union and its territory
Lecture 4 union and its territory
amanbishla1
 
Indian independence act
Indian independence actIndian independence act
Indian independence act
Sanket Gogoi
 
Union and its territory
Union and its territoryUnion and its territory
Union and its territory
ExamSprint
 
Government of India Act 1935
Government of India Act 1935Government of India Act 1935
Government of India Act 1935
Abdul Azim Akhtar
 
Lecture 3 preamble
Lecture 3   preambleLecture 3   preamble
Lecture 3 preamble
amanbishla1
 
1858 act
1858 act1858 act
Polity mcq
Polity mcqPolity mcq
Polity mcq
JHUTULLU
 
Polity governance UGC-NET PAPER-1 AS PER NEW UPDATED SYLLABUS
Polity governance UGC-NET  PAPER-1  AS PER NEW UPDATED SYLLABUS Polity governance UGC-NET  PAPER-1  AS PER NEW UPDATED SYLLABUS
Polity governance UGC-NET PAPER-1 AS PER NEW UPDATED SYLLABUS
DIwakar Rajput
 
Union territory and tribal councils
Union territory and tribal councilsUnion territory and tribal councils
Union territory and tribal councils
Shyam Rathod
 
Right to freedom
Right to freedomRight to freedom
Right to freedom
Anirban Mandal
 
Tamizzudin's Case
Tamizzudin's CaseTamizzudin's Case
Tamizzudin's Case
Fazal Akbar
 
Minto morley reforms
Minto morley reformsMinto morley reforms
Minto morley reforms
Amber Memon
 
Indian politics
Indian politicsIndian politics
Indian politics
Rekha Choudhary
 

What's hot (19)

Salient features of constitution of India
Salient features of constitution of IndiaSalient features of constitution of India
Salient features of constitution of India
 
Lecture 5 citizenship
Lecture 5   citizenshipLecture 5   citizenship
Lecture 5 citizenship
 
Constitution of india a living document
Constitution of india  a living documentConstitution of india  a living document
Constitution of india a living document
 
Salient features of indian constitutions.
Salient features of indian constitutions.Salient features of indian constitutions.
Salient features of indian constitutions.
 
Lecture 2 introduction to indian constitution
Lecture 2   introduction to indian constitutionLecture 2   introduction to indian constitution
Lecture 2 introduction to indian constitution
 
Lecture 1-historical-background
Lecture 1-historical-backgroundLecture 1-historical-background
Lecture 1-historical-background
 
Lecture 4 union and its territory
Lecture 4   union and its territoryLecture 4   union and its territory
Lecture 4 union and its territory
 
Indian independence act
Indian independence actIndian independence act
Indian independence act
 
Union and its territory
Union and its territoryUnion and its territory
Union and its territory
 
Government of India Act 1935
Government of India Act 1935Government of India Act 1935
Government of India Act 1935
 
Lecture 3 preamble
Lecture 3   preambleLecture 3   preamble
Lecture 3 preamble
 
1858 act
1858 act1858 act
1858 act
 
Polity mcq
Polity mcqPolity mcq
Polity mcq
 
Polity governance UGC-NET PAPER-1 AS PER NEW UPDATED SYLLABUS
Polity governance UGC-NET  PAPER-1  AS PER NEW UPDATED SYLLABUS Polity governance UGC-NET  PAPER-1  AS PER NEW UPDATED SYLLABUS
Polity governance UGC-NET PAPER-1 AS PER NEW UPDATED SYLLABUS
 
Union territory and tribal councils
Union territory and tribal councilsUnion territory and tribal councils
Union territory and tribal councils
 
Right to freedom
Right to freedomRight to freedom
Right to freedom
 
Tamizzudin's Case
Tamizzudin's CaseTamizzudin's Case
Tamizzudin's Case
 
Minto morley reforms
Minto morley reformsMinto morley reforms
Minto morley reforms
 
Indian politics
Indian politicsIndian politics
Indian politics
 

Similar to Indian polity

General Knowledge for clat 2015 - Sample Questions
General Knowledge for clat 2015 - Sample QuestionsGeneral Knowledge for clat 2015 - Sample Questions
General Knowledge for clat 2015 - Sample Questions
CLAT Preparation
 
100 mcq of constitution
100 mcq of constitution100 mcq of constitution
100 mcq of constitution
Sunil Bhardwaj
 
Indian polity
Indian polityIndian polity
Indian polity
cskumaran1
 
Rohingya answer 2
Rohingya answer 2Rohingya answer 2
Rohingya answer 2
ZahidManiyar
 
Lok sabha answer june 20 (1)
Lok sabha answer june 20 (1)Lok sabha answer june 20 (1)
Lok sabha answer june 20 (1)
ZahidManiyar
 
GK-14 _ Polity-2_Qs.pdf cuet revision must learn
GK-14 _ Polity-2_Qs.pdf cuet revision must learnGK-14 _ Polity-2_Qs.pdf cuet revision must learn
GK-14 _ Polity-2_Qs.pdf cuet revision must learn
nityagupta74
 
Indian constitution
Indian constitutionIndian constitution
Indian constitution
ABHISHEK KUMAR
 
Constutional Law I - session 10.pptx
Constutional Law I - session 10.pptxConstutional Law I - session 10.pptx
Constutional Law I - session 10.pptx
ketan349068
 
CLAT 2019 PG Question Paper
CLAT 2019 PG Question PaperCLAT 2019 PG Question Paper
CLAT 2019 PG Question Paper
VidhiWise
 
Slides15
Slides15Slides15
Slides15
Akmal Wasim
 
Citizenship Amendment Act 2019 FACTS & LIES
Citizenship Amendment Act 2019 FACTS & LIESCitizenship Amendment Act 2019 FACTS & LIES
Citizenship Amendment Act 2019 FACTS & LIES
Patriotic Indian Christian
 
List of all salient features of Indian constitution
 List of all salient features of Indian constitution  List of all salient features of Indian constitution
List of all salient features of Indian constitution
harishchandrasahani
 
CIP_PPT (1).ppt.dddddddddddddddddddddddddd
CIP_PPT (1).ppt.ddddddddddddddddddddddddddCIP_PPT (1).ppt.dddddddddddddddddddddddddd
CIP_PPT (1).ppt.dddddddddddddddddddddddddd
GooGle942495
 
One lakh General knowledge
One lakh General knowledgeOne lakh General knowledge
One lakh General knowledge
narayan changder
 
Fundamental Rights The Indian Nature
Fundamental Rights The Indian NatureFundamental Rights The Indian Nature
Fundamental Rights The Indian Nature
YogeshIJTSRD
 
civics.pptx
civics.pptxcivics.pptx
civics.pptx
NightmareKing
 
Ch 4 Basic Features of the Indian Constitution
Ch 4 Basic Features of the Indian ConstitutionCh 4 Basic Features of the Indian Constitution
Ch 4 Basic Features of the Indian Constitution
Teena9
 
Quiz on the constitution of india
Quiz on the constitution of indiaQuiz on the constitution of india
Quiz on the constitution of india
MikaPriya
 
Constitutionofindiaacts
ConstitutionofindiaactsConstitutionofindiaacts
Constitutionofindiaacts
SASWAT MAHAPATRA
 
Impact and its Aftermath of 18th Amend..pptx
Impact and its Aftermath of 18th Amend..pptxImpact and its Aftermath of 18th Amend..pptx
Impact and its Aftermath of 18th Amend..pptx
4rf5v8w8fb
 

Similar to Indian polity (20)

General Knowledge for clat 2015 - Sample Questions
General Knowledge for clat 2015 - Sample QuestionsGeneral Knowledge for clat 2015 - Sample Questions
General Knowledge for clat 2015 - Sample Questions
 
100 mcq of constitution
100 mcq of constitution100 mcq of constitution
100 mcq of constitution
 
Indian polity
Indian polityIndian polity
Indian polity
 
Rohingya answer 2
Rohingya answer 2Rohingya answer 2
Rohingya answer 2
 
Lok sabha answer june 20 (1)
Lok sabha answer june 20 (1)Lok sabha answer june 20 (1)
Lok sabha answer june 20 (1)
 
GK-14 _ Polity-2_Qs.pdf cuet revision must learn
GK-14 _ Polity-2_Qs.pdf cuet revision must learnGK-14 _ Polity-2_Qs.pdf cuet revision must learn
GK-14 _ Polity-2_Qs.pdf cuet revision must learn
 
Indian constitution
Indian constitutionIndian constitution
Indian constitution
 
Constutional Law I - session 10.pptx
Constutional Law I - session 10.pptxConstutional Law I - session 10.pptx
Constutional Law I - session 10.pptx
 
CLAT 2019 PG Question Paper
CLAT 2019 PG Question PaperCLAT 2019 PG Question Paper
CLAT 2019 PG Question Paper
 
Slides15
Slides15Slides15
Slides15
 
Citizenship Amendment Act 2019 FACTS & LIES
Citizenship Amendment Act 2019 FACTS & LIESCitizenship Amendment Act 2019 FACTS & LIES
Citizenship Amendment Act 2019 FACTS & LIES
 
List of all salient features of Indian constitution
 List of all salient features of Indian constitution  List of all salient features of Indian constitution
List of all salient features of Indian constitution
 
CIP_PPT (1).ppt.dddddddddddddddddddddddddd
CIP_PPT (1).ppt.ddddddddddddddddddddddddddCIP_PPT (1).ppt.dddddddddddddddddddddddddd
CIP_PPT (1).ppt.dddddddddddddddddddddddddd
 
One lakh General knowledge
One lakh General knowledgeOne lakh General knowledge
One lakh General knowledge
 
Fundamental Rights The Indian Nature
Fundamental Rights The Indian NatureFundamental Rights The Indian Nature
Fundamental Rights The Indian Nature
 
civics.pptx
civics.pptxcivics.pptx
civics.pptx
 
Ch 4 Basic Features of the Indian Constitution
Ch 4 Basic Features of the Indian ConstitutionCh 4 Basic Features of the Indian Constitution
Ch 4 Basic Features of the Indian Constitution
 
Quiz on the constitution of india
Quiz on the constitution of indiaQuiz on the constitution of india
Quiz on the constitution of india
 
Constitutionofindiaacts
ConstitutionofindiaactsConstitutionofindiaacts
Constitutionofindiaacts
 
Impact and its Aftermath of 18th Amend..pptx
Impact and its Aftermath of 18th Amend..pptxImpact and its Aftermath of 18th Amend..pptx
Impact and its Aftermath of 18th Amend..pptx
 

More from DEEPAK CHAUHAN

Ocean Water and Currents PPT LESSON
Ocean Water and Currents PPT LESSONOcean Water and Currents PPT LESSON
Ocean Water and Currents PPT LESSON
DEEPAK CHAUHAN
 
UPSC test Series schedule.pdf
UPSC test Series schedule.pdfUPSC test Series schedule.pdf
UPSC test Series schedule.pdf
DEEPAK CHAUHAN
 
UPSC Current affairs 04-09-2023 (Hindi).docx
UPSC Current affairs 04-09-2023 (Hindi).docxUPSC Current affairs 04-09-2023 (Hindi).docx
UPSC Current affairs 04-09-2023 (Hindi).docx
DEEPAK CHAUHAN
 
UPSC Current Affairs 02-09-2023 (Hindi)
UPSC Current Affairs 02-09-2023 (Hindi)UPSC Current Affairs 02-09-2023 (Hindi)
UPSC Current Affairs 02-09-2023 (Hindi)
DEEPAK CHAUHAN
 
06-09-2023.pdf
06-09-2023.pdf06-09-2023.pdf
06-09-2023.pdf
DEEPAK CHAUHAN
 
04-09-23.pdf
04-09-23.pdf04-09-23.pdf
04-09-23.pdf
DEEPAK CHAUHAN
 
5-09-23.pdf
5-09-23.pdf5-09-23.pdf
5-09-23.pdf
DEEPAK CHAUHAN
 
02-09-23.pdf
02-09-23.pdf02-09-23.pdf
02-09-23.pdf
DEEPAK CHAUHAN
 
07-09-2023.pdf
07-09-2023.pdf07-09-2023.pdf
07-09-2023.pdf
DEEPAK CHAUHAN
 
13-9-23.pdf (Daily Current Affairs)
13-9-23.pdf (Daily Current Affairs)13-9-23.pdf (Daily Current Affairs)
13-9-23.pdf (Daily Current Affairs)
DEEPAK CHAUHAN
 
African Union (AU)
African Union (AU)African Union (AU)
African Union (AU)
DEEPAK CHAUHAN
 
9-9-23.pdf
9-9-23.pdf9-9-23.pdf
9-9-23.pdf
DEEPAK CHAUHAN
 
8-9-23.pdf
8-9-23.pdf8-9-23.pdf
8-9-23.pdf
DEEPAK CHAUHAN
 
11-9-23.pdf
11-9-23.pdf11-9-23.pdf
11-9-23.pdf
DEEPAK CHAUHAN
 
UPSC Mains Syllabus IASNEXT.pdf
UPSC Mains Syllabus IASNEXT.pdfUPSC Mains Syllabus IASNEXT.pdf
UPSC Mains Syllabus IASNEXT.pdf
DEEPAK CHAUHAN
 
UPSC Mains Syllabus IASNEXT.pdf
UPSC Mains Syllabus IASNEXT.pdfUPSC Mains Syllabus IASNEXT.pdf
UPSC Mains Syllabus IASNEXT.pdf
DEEPAK CHAUHAN
 
UP PCS MAINS SYLLABUS.pdf
UP PCS MAINS SYLLABUS.pdfUP PCS MAINS SYLLABUS.pdf
UP PCS MAINS SYLLABUS.pdf
DEEPAK CHAUHAN
 
UP PCS PRELIMS AND MAINS SYLLABUS.pdf
UP PCS PRELIMS AND MAINS SYLLABUS.pdfUP PCS PRELIMS AND MAINS SYLLABUS.pdf
UP PCS PRELIMS AND MAINS SYLLABUS.pdf
DEEPAK CHAUHAN
 
Geography notes by ias next
Geography notes by ias nextGeography notes by ias next
Geography notes by ias next
DEEPAK CHAUHAN
 
Foundation course detail
Foundation course detailFoundation course detail
Foundation course detail
DEEPAK CHAUHAN
 

More from DEEPAK CHAUHAN (20)

Ocean Water and Currents PPT LESSON
Ocean Water and Currents PPT LESSONOcean Water and Currents PPT LESSON
Ocean Water and Currents PPT LESSON
 
UPSC test Series schedule.pdf
UPSC test Series schedule.pdfUPSC test Series schedule.pdf
UPSC test Series schedule.pdf
 
UPSC Current affairs 04-09-2023 (Hindi).docx
UPSC Current affairs 04-09-2023 (Hindi).docxUPSC Current affairs 04-09-2023 (Hindi).docx
UPSC Current affairs 04-09-2023 (Hindi).docx
 
UPSC Current Affairs 02-09-2023 (Hindi)
UPSC Current Affairs 02-09-2023 (Hindi)UPSC Current Affairs 02-09-2023 (Hindi)
UPSC Current Affairs 02-09-2023 (Hindi)
 
06-09-2023.pdf
06-09-2023.pdf06-09-2023.pdf
06-09-2023.pdf
 
04-09-23.pdf
04-09-23.pdf04-09-23.pdf
04-09-23.pdf
 
5-09-23.pdf
5-09-23.pdf5-09-23.pdf
5-09-23.pdf
 
02-09-23.pdf
02-09-23.pdf02-09-23.pdf
02-09-23.pdf
 
07-09-2023.pdf
07-09-2023.pdf07-09-2023.pdf
07-09-2023.pdf
 
13-9-23.pdf (Daily Current Affairs)
13-9-23.pdf (Daily Current Affairs)13-9-23.pdf (Daily Current Affairs)
13-9-23.pdf (Daily Current Affairs)
 
African Union (AU)
African Union (AU)African Union (AU)
African Union (AU)
 
9-9-23.pdf
9-9-23.pdf9-9-23.pdf
9-9-23.pdf
 
8-9-23.pdf
8-9-23.pdf8-9-23.pdf
8-9-23.pdf
 
11-9-23.pdf
11-9-23.pdf11-9-23.pdf
11-9-23.pdf
 
UPSC Mains Syllabus IASNEXT.pdf
UPSC Mains Syllabus IASNEXT.pdfUPSC Mains Syllabus IASNEXT.pdf
UPSC Mains Syllabus IASNEXT.pdf
 
UPSC Mains Syllabus IASNEXT.pdf
UPSC Mains Syllabus IASNEXT.pdfUPSC Mains Syllabus IASNEXT.pdf
UPSC Mains Syllabus IASNEXT.pdf
 
UP PCS MAINS SYLLABUS.pdf
UP PCS MAINS SYLLABUS.pdfUP PCS MAINS SYLLABUS.pdf
UP PCS MAINS SYLLABUS.pdf
 
UP PCS PRELIMS AND MAINS SYLLABUS.pdf
UP PCS PRELIMS AND MAINS SYLLABUS.pdfUP PCS PRELIMS AND MAINS SYLLABUS.pdf
UP PCS PRELIMS AND MAINS SYLLABUS.pdf
 
Geography notes by ias next
Geography notes by ias nextGeography notes by ias next
Geography notes by ias next
 
Foundation course detail
Foundation course detailFoundation course detail
Foundation course detail
 

Recently uploaded

LAND USE LAND COVER AND NDVI OF MIRZAPUR DISTRICT, UP
LAND USE LAND COVER AND NDVI OF MIRZAPUR DISTRICT, UPLAND USE LAND COVER AND NDVI OF MIRZAPUR DISTRICT, UP
LAND USE LAND COVER AND NDVI OF MIRZAPUR DISTRICT, UP
RAHUL
 
The simplified electron and muon model, Oscillating Spacetime: The Foundation...
The simplified electron and muon model, Oscillating Spacetime: The Foundation...The simplified electron and muon model, Oscillating Spacetime: The Foundation...
The simplified electron and muon model, Oscillating Spacetime: The Foundation...
RitikBhardwaj56
 
বাংলাদেশ অর্থনৈতিক সমীক্ষা (Economic Review) ২০২৪ UJS App.pdf
বাংলাদেশ অর্থনৈতিক সমীক্ষা (Economic Review) ২০২৪ UJS App.pdfবাংলাদেশ অর্থনৈতিক সমীক্ষা (Economic Review) ২০২৪ UJS App.pdf
বাংলাদেশ অর্থনৈতিক সমীক্ষা (Economic Review) ২০২৪ UJS App.pdf
eBook.com.bd (প্রয়োজনীয় বাংলা বই)
 
S1-Introduction-Biopesticides in ICM.pptx
S1-Introduction-Biopesticides in ICM.pptxS1-Introduction-Biopesticides in ICM.pptx
S1-Introduction-Biopesticides in ICM.pptx
tarandeep35
 
The History of Stoke Newington Street Names
The History of Stoke Newington Street NamesThe History of Stoke Newington Street Names
The History of Stoke Newington Street Names
History of Stoke Newington
 
Advanced Java[Extra Concepts, Not Difficult].docx
Advanced Java[Extra Concepts, Not Difficult].docxAdvanced Java[Extra Concepts, Not Difficult].docx
Advanced Java[Extra Concepts, Not Difficult].docx
adhitya5119
 
A Independência da América Espanhola LAPBOOK.pdf
A Independência da América Espanhola LAPBOOK.pdfA Independência da América Espanhola LAPBOOK.pdf
A Independência da América Espanhola LAPBOOK.pdf
Jean Carlos Nunes Paixão
 
Chapter 4 - Islamic Financial Institutions in Malaysia.pptx
Chapter 4 - Islamic Financial Institutions in Malaysia.pptxChapter 4 - Islamic Financial Institutions in Malaysia.pptx
Chapter 4 - Islamic Financial Institutions in Malaysia.pptx
Mohd Adib Abd Muin, Senior Lecturer at Universiti Utara Malaysia
 
Your Skill Boost Masterclass: Strategies for Effective Upskilling
Your Skill Boost Masterclass: Strategies for Effective UpskillingYour Skill Boost Masterclass: Strategies for Effective Upskilling
Your Skill Boost Masterclass: Strategies for Effective Upskilling
Excellence Foundation for South Sudan
 
How to Add Chatter in the odoo 17 ERP Module
How to Add Chatter in the odoo 17 ERP ModuleHow to Add Chatter in the odoo 17 ERP Module
How to Add Chatter in the odoo 17 ERP Module
Celine George
 
PCOS corelations and management through Ayurveda.
PCOS corelations and management through Ayurveda.PCOS corelations and management through Ayurveda.
PCOS corelations and management through Ayurveda.
Dr. Shivangi Singh Parihar
 
Azure Interview Questions and Answers PDF By ScholarHat
Azure Interview Questions and Answers PDF By ScholarHatAzure Interview Questions and Answers PDF By ScholarHat
Azure Interview Questions and Answers PDF By ScholarHat
Scholarhat
 
Natural birth techniques - Mrs.Akanksha Trivedi Rama University
Natural birth techniques - Mrs.Akanksha Trivedi Rama UniversityNatural birth techniques - Mrs.Akanksha Trivedi Rama University
Natural birth techniques - Mrs.Akanksha Trivedi Rama University
Akanksha trivedi rama nursing college kanpur.
 
clinical examination of hip joint (1).pdf
clinical examination of hip joint (1).pdfclinical examination of hip joint (1).pdf
clinical examination of hip joint (1).pdf
Priyankaranawat4
 
Digital Artefact 1 - Tiny Home Environmental Design
Digital Artefact 1 - Tiny Home Environmental DesignDigital Artefact 1 - Tiny Home Environmental Design
Digital Artefact 1 - Tiny Home Environmental Design
amberjdewit93
 
Walmart Business+ and Spark Good for Nonprofits.pdf
Walmart Business+ and Spark Good for Nonprofits.pdfWalmart Business+ and Spark Good for Nonprofits.pdf
Walmart Business+ and Spark Good for Nonprofits.pdf
TechSoup
 
How to Make a Field Mandatory in Odoo 17
How to Make a Field Mandatory in Odoo 17How to Make a Field Mandatory in Odoo 17
How to Make a Field Mandatory in Odoo 17
Celine George
 
How to Manage Your Lost Opportunities in Odoo 17 CRM
How to Manage Your Lost Opportunities in Odoo 17 CRMHow to Manage Your Lost Opportunities in Odoo 17 CRM
How to Manage Your Lost Opportunities in Odoo 17 CRM
Celine George
 
Executive Directors Chat Leveraging AI for Diversity, Equity, and Inclusion
Executive Directors Chat  Leveraging AI for Diversity, Equity, and InclusionExecutive Directors Chat  Leveraging AI for Diversity, Equity, and Inclusion
Executive Directors Chat Leveraging AI for Diversity, Equity, and Inclusion
TechSoup
 
How to Build a Module in Odoo 17 Using the Scaffold Method
How to Build a Module in Odoo 17 Using the Scaffold MethodHow to Build a Module in Odoo 17 Using the Scaffold Method
How to Build a Module in Odoo 17 Using the Scaffold Method
Celine George
 

Recently uploaded (20)

LAND USE LAND COVER AND NDVI OF MIRZAPUR DISTRICT, UP
LAND USE LAND COVER AND NDVI OF MIRZAPUR DISTRICT, UPLAND USE LAND COVER AND NDVI OF MIRZAPUR DISTRICT, UP
LAND USE LAND COVER AND NDVI OF MIRZAPUR DISTRICT, UP
 
The simplified electron and muon model, Oscillating Spacetime: The Foundation...
The simplified electron and muon model, Oscillating Spacetime: The Foundation...The simplified electron and muon model, Oscillating Spacetime: The Foundation...
The simplified electron and muon model, Oscillating Spacetime: The Foundation...
 
বাংলাদেশ অর্থনৈতিক সমীক্ষা (Economic Review) ২০২৪ UJS App.pdf
বাংলাদেশ অর্থনৈতিক সমীক্ষা (Economic Review) ২০২৪ UJS App.pdfবাংলাদেশ অর্থনৈতিক সমীক্ষা (Economic Review) ২০২৪ UJS App.pdf
বাংলাদেশ অর্থনৈতিক সমীক্ষা (Economic Review) ২০২৪ UJS App.pdf
 
S1-Introduction-Biopesticides in ICM.pptx
S1-Introduction-Biopesticides in ICM.pptxS1-Introduction-Biopesticides in ICM.pptx
S1-Introduction-Biopesticides in ICM.pptx
 
The History of Stoke Newington Street Names
The History of Stoke Newington Street NamesThe History of Stoke Newington Street Names
The History of Stoke Newington Street Names
 
Advanced Java[Extra Concepts, Not Difficult].docx
Advanced Java[Extra Concepts, Not Difficult].docxAdvanced Java[Extra Concepts, Not Difficult].docx
Advanced Java[Extra Concepts, Not Difficult].docx
 
A Independência da América Espanhola LAPBOOK.pdf
A Independência da América Espanhola LAPBOOK.pdfA Independência da América Espanhola LAPBOOK.pdf
A Independência da América Espanhola LAPBOOK.pdf
 
Chapter 4 - Islamic Financial Institutions in Malaysia.pptx
Chapter 4 - Islamic Financial Institutions in Malaysia.pptxChapter 4 - Islamic Financial Institutions in Malaysia.pptx
Chapter 4 - Islamic Financial Institutions in Malaysia.pptx
 
Your Skill Boost Masterclass: Strategies for Effective Upskilling
Your Skill Boost Masterclass: Strategies for Effective UpskillingYour Skill Boost Masterclass: Strategies for Effective Upskilling
Your Skill Boost Masterclass: Strategies for Effective Upskilling
 
How to Add Chatter in the odoo 17 ERP Module
How to Add Chatter in the odoo 17 ERP ModuleHow to Add Chatter in the odoo 17 ERP Module
How to Add Chatter in the odoo 17 ERP Module
 
PCOS corelations and management through Ayurveda.
PCOS corelations and management through Ayurveda.PCOS corelations and management through Ayurveda.
PCOS corelations and management through Ayurveda.
 
Azure Interview Questions and Answers PDF By ScholarHat
Azure Interview Questions and Answers PDF By ScholarHatAzure Interview Questions and Answers PDF By ScholarHat
Azure Interview Questions and Answers PDF By ScholarHat
 
Natural birth techniques - Mrs.Akanksha Trivedi Rama University
Natural birth techniques - Mrs.Akanksha Trivedi Rama UniversityNatural birth techniques - Mrs.Akanksha Trivedi Rama University
Natural birth techniques - Mrs.Akanksha Trivedi Rama University
 
clinical examination of hip joint (1).pdf
clinical examination of hip joint (1).pdfclinical examination of hip joint (1).pdf
clinical examination of hip joint (1).pdf
 
Digital Artefact 1 - Tiny Home Environmental Design
Digital Artefact 1 - Tiny Home Environmental DesignDigital Artefact 1 - Tiny Home Environmental Design
Digital Artefact 1 - Tiny Home Environmental Design
 
Walmart Business+ and Spark Good for Nonprofits.pdf
Walmart Business+ and Spark Good for Nonprofits.pdfWalmart Business+ and Spark Good for Nonprofits.pdf
Walmart Business+ and Spark Good for Nonprofits.pdf
 
How to Make a Field Mandatory in Odoo 17
How to Make a Field Mandatory in Odoo 17How to Make a Field Mandatory in Odoo 17
How to Make a Field Mandatory in Odoo 17
 
How to Manage Your Lost Opportunities in Odoo 17 CRM
How to Manage Your Lost Opportunities in Odoo 17 CRMHow to Manage Your Lost Opportunities in Odoo 17 CRM
How to Manage Your Lost Opportunities in Odoo 17 CRM
 
Executive Directors Chat Leveraging AI for Diversity, Equity, and Inclusion
Executive Directors Chat  Leveraging AI for Diversity, Equity, and InclusionExecutive Directors Chat  Leveraging AI for Diversity, Equity, and Inclusion
Executive Directors Chat Leveraging AI for Diversity, Equity, and Inclusion
 
How to Build a Module in Odoo 17 Using the Scaffold Method
How to Build a Module in Odoo 17 Using the Scaffold MethodHow to Build a Module in Odoo 17 Using the Scaffold Method
How to Build a Module in Odoo 17 Using the Scaffold Method
 

Indian polity

  • 1. Indian Polity For more info: www.iasnext.com Page 1 Ph. :0522 4241011/9454721860 1. The Cabinet Mission to India was headed by : (a) Stafford Cripps (b) A.V. Alexander (c) Lord Pethick Lawrence (d) Hugh Gaitskell 2. The Constituent Assembly for undivided India first met on (a) 6th December, 1946 (b) 9th December, 1946 (c) 20th February, 1947 (d) 3rd June, 1947 3. When the Constituent Assembly for the Dominion of India reassembled on 31st October, 1947, its reduced membership was: (a) 299 (b) 311 (c) 319 (d) 331 4. How long did the Constituent Assembly take to finally pass the Constitution? (a) about 6 months in 1949 (b) exactly a year since Nov 26, 1948 (c) about 2 years since Aug 15, 1947 (d) about 3 years since Dec 9, 1946 5. Who presided over the inaugural meeting of the Constituent Assembly of India? (a) Sachidananda Sinha (b) B. R. Ambedkar (c) Dr. Rajendra Prasad (d) P. Upendra 6. The Constitution of India was enacted by a Constituent Assembly set up : (a) under the Indian Independence Act, 1947 (b) under the Cabinet Mission Plan, 1946 (c) through a resolution of the provisional government (d) by the Indian National Congress 7. Who among the following was the Chairman of the Drafting Committee of the Indian Constitution? (a) Rajendra Prasad (b) Tej Bahadur Sapru (c) C. Rajagopalachari (d) B. R. Ambedkar 8. Match the following: Constitutional Provisions Sources A. Federation 1. USA B. Parliamentary 2. Britain C. Directive Principle 3. Canada D. Fundamental Rights 4. Ireland A B C D (a) 2 1 4 1 (b) 2 3 1 4 (c) 3 4 2 1 (d) 3 2 4 1 9. Which of the following features and their source is wrongly matched? (a) Judicial review – British practice (b) Concurrent List – Australian Constitution (c) Directive Principles – Irish Constitution (d) Fundamental Rights – US Constitution 10. Which Constitution has inspired the Indian concepts of rule of law, parliamentary system and law-making procedure? (a) US Constitution (b) British Constitution (c) Canadian Constitution (d) All of the above 11. Which of the following emerges clearly from the Preamble? 1. When the Constitution was enacted. 2. The ideals that were to be achieved. 3. The system of government 4. The source of authority (a) II, III and IV (b) I and II (c) I, II and III (d) I, II, III and IV 12. Which one of the following statements regarding the Preamble is correct? (a) It is not enforceable in a court of law (b) the Supreme Court has recently ruled that it is not a part of the Constitution (c) It has been amended twice (d) All the above are correct 13. Match the following: Constitutional Features Sources A. Rule of Law 1. German Constitution B. Directive Principles 2. Australian Constitution C. Concurrent List 3. Irish Constitution D. Suspension of 4. British Constitution Fundamental Rights during Emergency A B C D (a) 4 2 1 3 (b) 4 3 2 1 (c) 2 3 4 1 (d) 2 1 4 3 14. Which of the following features does the Indian Constitution borrow from the Weimar Constitution of Germany? (a) The idea of a federation with a strong centre (b) The method of presidential elections (c) The Emergency powers (d) Provisions concerning the suspension of fundamental rights during National Emergency 15. The Constitution of India is designed to work as a unitary government: (a) in normal times (b) in times of Emergency (c) when the Parliament so desires (d) at the will of the President 16. Which of the following features of the Constitution of India does not resemble the American Constitution? (a) Written Constitution (b) Federal form of government (c) Fundamental Rights (d) Parliamentary form of government 17. Which of the following statements regarding the acquisition of Indian citizenship is correct ? (a) If citizenship is to be acquired by registration, six months residence in India is essential.
  • 2. Indian Polity For more info: www.iasnext.com Page 2 Ph. :0522 4241011/9454721860 (b) If citizenship is to be acquired by naturalisation the person concerned must have resided in India for 5 years. (c) If born in India, only that person can acquire citizenship at least one of whose parents is an Indian. (d) Rules regarding the citizenship of India shall not be applicable to Jammu and Kashmir. 18. According to the Citizenship Act, 1955, Indian citizenship cannot be acquired by: (a) children of the diplomatic personnel or alien enemies born in India on or after 26th January, 1950 (b) children born of Indian citizens abroad (c) incorporation of territory not forming part of India at the commencement of the Constitution (d) naturalization 19. Details on how citizenship may be acquired and terminated are enumerated in: (a) Part II of the Constitution (b) the Citizenship Act, 1955 (c) Schedule I of the Constitution (d) various enactments by Parliament 20. Which of the following is/are included in the Directive Principles of State Policy? [IAS 2008] 1. Prohibition of traffic in human beings and forced labour 2. Prohibition of consumption except for medicinal purposes of intoxicating drinks and of other drugs which are injurious to health. (a) 1 only (b) 2 only (c) Both 1 and 2 (d) Neither 1 nor 2 21. The President of the Indian Union has the same constitutional authority as the: (a) British Monarch (b) President of USA (c) President of Egypt (d) President of Russia 22. Article 6 enshrines the provision conferring Rights of citizenship of certain persons who have migrated to India from Pakistan. Which one of the following provisions is incorrect? (a) He or either of his parents or any of his grand- parents was born in India as defined in the Government of India Act, 1935 (b) Such person has so migrated before 19 July, 1948 (c) Such person has so migrated on or after the 19 July, 1948, he has been registered as a citizen (d) Such person has got married in India 23. Which Schedule of the Constitution of India contains special provisions for the administration and control of Scheduled Areas in several States? (a) Third (b) Fifth (c) Seventh (d) Ninth 24. Indian citizenship of a person can be terminated if: 1. a person voluntarily acquires the citizenship of some other country. 2. a person who has become a citizen through registration is sentenced to imprisonment for not less than 10 years within five years of his registration. 3. the Government of India is satisfied that citizenship was obtained by fraud. 4. a person who is a citizen by birth indulges in trade with an enemy country during war. (a) I and III (b) I, II and III (c) I, III and IV (d) I, II, III and IV 25. Citizenship provisions are enshrined in the Constitution in : (a) Part II (b) Part II, Articles 5-11 (c) Part II, Articles 5-6 (d) Part I, Articles 5-11 26. As far as Armed Forces are concerned, the fundamental rights• granted under Articles 14 and 19 of the Constitution are: (a) not available at all (b) available to armed forces but not to other forces (c) available only at the discretion of the chief of army staff (d) available only according to law made by Parliament 27. Which of the following Articles describes about the person voluntarily acquiring citizenship of a foreign state not to be citizens? (a) Article 5 (b) Article 7 (c) Article 8 (d) Article 9 28. Which of the following statements is/are not indicative of the difference between Fundamental Rights and Directive Principles? I. Directive Principles are aimed at promoting social welfare, while Fundamental Rights are for protecting individuals from State encroachment. II. Fundamental Rights are limitations on State action, while Directive Principles are positive instructions for the Government to work towards a just socioeconomic order. III. Fundamental Rights were included in the original Constitution, but Directive Principles were added by the first Amendment. IV. Fundamental Rights are amendable, but Directive Principles cannot be amended. (a) I and II (b) II and III (c) III and IV (d) I, II and III 29. Consider the following statements in respect of financial emergency under Article 360 of the Constitution of India: 1. A proclamation of financial emergency issued shall cease to operate at the expiration of two months, unless before the expiration of that period it has been approved by the resolutions of both Houses of Parliament. 2. If any proclamation of financial emergency Is in operation. it is competent for the President of India to issue directions for the reduction of salaries and allowances of all or any class of persons serving in connection with the affairs of the Union but
  • 3. Indian Polity For more info: www.iasnext.com Page 3 Ph. :0522 4241011/9454721860 excluding the Judges of the Supreme Court and the High Courts. Which of the statements given above is/are correct? (a) 1 only (b) 2 only (c) Both 1 and 2 (d) Neither 1 nor 2 30. Part V of the Constitution deals with: I. Union Executive II. Parliament III. Supreme Court and High Courts IV. Comptroller and Auditor-General (a) I and II (b) I, II and III (c) I only (d) I, II and IV 31. Match the following - Articles of the Constitution of India 1. Article 14 2. Article 15 3. Article 16 4. Article 17 Provision 1. The State shall not discriminate against any citizen on grounds only of religion, race, caste, sex, place of birth or any of them 2. The State shall not deny to any person equality before the law or the equal protection of laws within the territory of India 3. 'Untouchability' is abolished and its practice in any form is forbidden 33. There shall be equality of opportunity for all citizens in matters relating to employment or appointment to any office under the State A B C D (a) 2 4 1 3 (b) 3 1 4 2 (c) 2 1 4 3 (d) 2 4 1 2 32. The Governor recommends to the President of India that breakdown of Constitutional machinery in the State is imminent. The President makes a proclamation under Article 356. The action of the President: (a) cannot be reviewed as the President is the sole judge of his emergency powers (b) cannot be reviewed as the Constitution bars the courts from reviewing political actions (c) can be reviewed as it goes against Fundamental Rights (d) can be reviewed as it is malafide 33. Provisions of having a UPSC and Public Service Commision for States are enshrined in : (a) Part XIV, Chapter II, Articles 315-324 (b) Part XIV, Chapter I, Articles 308-323 (c) Part XIV, Chapter II, Articles 315-323 (d) Part XIV, Chapter I, Articles 308-318 34. Match the following: A. Part I 1. Fundamental Rights B. Part III 2. Panchayati Raj C. Part IX 3. Citizenship D. Part II 4. The Union and its Territory A B C D (a) 4 2 3 1 (b) 3 1 2 4 (c) 2 3 1 3 (d) 4 1 2 3 35. Match the following: A. Article 61 1. Removal of Deputy Chairman of Rajya Sabha B. Article 67 2. Impeachement of President C. Article 94 3. Removal of Vice President D. Article 90 4. Removal of Speaker A B C D (a) 2 3 4 1 (b) 1 3 4 1 (c) 2 1 4 3 (d) 3 4 2 1 36. Match the following: A. Abolition of Untouchability 1. Article 24 B. Abolition of Titles 2. Article 23 C. Prohibition of Child labour 3. Article 17 D. Prohibition of Traffic in 4. Article 18 human beings A B C D (a) 3 4 2 1 (b) 2 4 1 3 (c) 3 4 1 2 (d) 1 3 2 4 37. During financial emergency, the President can: 1. ask the states to reduce the salaries arid allowances of all or any class of persons serving in connection with the affairs of the State 2. ask the states to reserve money bills passed by the state legislature for his consideration 3. issue directions to states on financial matters 4. issue directions for the reduction of salaries and allowances of persons serving in connection with the affairs of the Union (a) I, II and III (b) I, III and IV (c) II, III and IV (d) I, II, III and IV 38. During a proclamation of emergency due to the breakdown of constitutional machinery in a State the President can I. assume all powers vested in and exercisable by the Governor II. declare that the powers of the State Legislature shall be exercised by Parliament III. assume certain powers of the High Courts IV. suspend by order any or all Fundamental Rights except those under Articles 20 and 21 (a) I and II (b) I and III (c) II, III and IV (d) II and IV 39. The value of a vote of a Member of Parliament for the election of the President is determined by dividing the: (a) nation's population as per the latest census by the number of Lok Sabha members (b) nation's population as per the latest census by the total strength of the two Houses of Paliament
  • 4. Indian Polity For more info: www.iasnext.com Page 4 Ph. :0522 4241011/9454721860 (c) the total value of votes of members of all the State Legislative Assemblies by the elected Members of the two Houses of Parliament (d) particular State's population as per the latest census by the number of Members of Parliament elected from that State 40. Who among the following' are appointed by the President of India? I. Chairman, Finance Commission II. Deputy Chairman, Planning Commission III. Chief Minister of a Union Territory (a) I only (b) I and II (c) I and III (d) II and III 41. Which of the following is/are resultant of a proclamation of National Emergency because of war? I. The Union Government can give directions to the States about how the executive power of theState is to be exercised. II. The Fundamental Rights stand automatically suspended. III. The State Legislature is suspended. IV. Parliament can make laws with respect to any subject in the State List. (a) I, II and IV (b) I, III and IV (c) I only (d) I and IV 42. In case of proclamation of emergency on grounds of war or external aggression: (a) all Fundamental Rights will be automatically suspended (b) the right to move a court for enforcement of any Fundamental Right is suspended (c) the President may order the suspension of enforcement of any Fundamental Right except Article 20-21 (d) Parliament may authorise suspension of all Fundamental Rights 43. The President of India is: (a) Commander-in-Chief of Defence Forces (b) Supreme Commander of Armed Forces (c) Head of the Government (d) Supreme Commander of Defence Forces of the Union and Executive Head of the Union 44. Which of the following statements regarding the pardoning powers of the President is/are not correct? 1. He has the pardoning power in respect of sentence by court martial. 2. He can grant reprieve and respite in case of punishment for an offence against any law of the land, Union or State. 3. He alone can pardon a sentence of death. 4. His exercise of the power of pardon is open to judicial review. (a) I and III (b) II and III (c) I and IV (d) II and IV 45. The President of India is vested with ordinance making power by Article 126, He exercises this power: (a) when Lok Sabha is not in session and circumstances .exist which render it necessary for the President to take immediate action (b) when Council of States is not in session and the President is satisfied that circumstances exist which render it necessary for him to take immediate action (c) when both Houses of Parliament are not in session and President is satisfied that circumstances exist which made it necessary for him to take the immediate action (d) in all the above circumstances 46. Consider the following statements about the powers of the President of India: 1. The President can direct that any matter on which decision has been taken by a Minister should be placed before the Council of Ministers. 2. The President can call all information relating to proposals for legislation. 3. The President has the right to address and send messages to either House of the Parliament. 4. All decisions of the Council of Ministers relating to the administration fo the Union must be communicated to the President. Which of the statements given above are correct? (a) 1, 2 and 3 only (b) 1 and 3 only (c) 2 and 4 only (d) 1, 2, 3 and 4 47. The Prime Minister, at the time of the appointment: 1. need not necessarily be a member of one of the Houses of Parliament but must become a member of one of the Houses within six months. 2. need not necessarily be a member of one of the. Houses of Parliament but must become a member of the Lok Sabha within six months. 3. must be either a nominated or elected member of one of the Houses of Parliament. 4. must be an elected member of only Lok Sabha. (a) I only (b) I and III (c) II only (d) IV only 48. Consider the following Vice-President of India: 1. V.V. Giri 2. M. Hidayatullah 3. B.D. Jatti 4. G.S. Pathak Which one of the following is the correct chronology of their tenures? (a) 1-4-3-2 (b) 2-1-3-4 (c) 3-2-1-4 (d) 4-1-3-2 49. Who among the following have held the office of the Vice-President of India? 1. Mohammad Hidayatullah 2. Fakhruddin Ali Ahmed 3. Neelam Sanjiva Reddy 4. Shankar Dayal Sharma (a) 1, 2, 3 and 4 (b) 1 and 4 (c) 2 and 3 (d) 3 and 4
  • 5. Indian Polity For more info: www.iasnext.com Page 5 Ph. :0522 4241011/9454721860 50. Which of the following committees exist only in the Lok Sabha? 1. Business Advisory Committee. 2. Committee on Private Members Bill and Resolutions. 3. Committee on Petitions. 4. Committee on Assurances. 5. Estimates Committee. (a) I, II and III (b) III and IV (c) II and V (d) II, IV and V 51. Each member of Parliament who participates in the Presidential election is entitled to cast as many votes as are obtained by dividing the total number of votes of the Legislative Assemblies of all the States by the total number of elected members of the two Houses of Parliament. This ensures: (a) parity between the voting strengths of the States and the Parliament (b) parity among the States (c) uniformity of representation of the different States (d) All the above 52. What is the minimum percentage of seats a party should get to be recognised as the opposition party in the legislature? (a) 20% (b) 15% (c) 10% (d) No such limit 53. The maximum number of representatives of the States in Lok Sabha is : (a) 525 (b) 530 (c) 545 (d) 550 54. Match the following: Parliamentary Terms A. Starred Question B. Unstarred Question C. Short Notice Question Meaning 1. One asked by a member on matters of public importance of an urgent nature 2. One for which the Concerned minister has to lay on the table a written answer 3. One for which an oral answer is required to be given by a minister on the floor of the House A B C (a) 1 2 3 (b) 2 1 3 (c) 3 1 2 (d) 3 2 1 55. Who among the following was never the Lok Sabha Speaker? (a) K.V.K. Sundaram (b) G.S. Dhillon (c) Balirarn Bhagat (d) Hukarn Singh 56. The Union Territories get representation in: (a) Lok Sabha (b) Rajya Sabha (c) both Houses of Parliament (d) None of the above 57. Which of the following statements regarding the Esstimates Committee are correct? 1. No member of the Rajya Sabha is associated with it. 2. It has twenty members. 3. Its members are elected in accordance with the system of proportional representation for a period of one year. 4. The Speaker nominates one of its members to be its Chairman. (a) I, II and III (b) I, III and IV (c) I, II and IV (d) II, III and IV 58. Match the following: A. Business Advisory Committee 1. Looks into the mode of public expenditure B. Select Committee 2. Examines the veracity of budget estimates C. Estimates Committee 3. Considers a bill and submits to the House a report on it D. Public Accounts Committee 4. Prepares time table for the whole session A B C D (a) 1 2 3 4 (b) 2 3 4 1 (c) 3 1 2 4 (d) 4 3 2 1 59. Which of the following is the correct sequence of the grades of officers in the Central Secretariat? 1. Secretary 2. Additional Secretary 3. Joint Secretary 4. Deputy Secretary 5. Under Secretary 6. Director (a) 1, 2, 3, 6, 4, 5 (b) 2, 1, 3, 5, 4, 6 (c) 6, 4, 3, 5, 2, 1 (d) 1, 5, 4, 2, 6, 3 60. An Appropriation Bill: I. is necessary to draw money from the Consolidated Fund of India. II. cannot be amended to vary the amount of any charged expenditure. III. Includes only the expenditure charged on the Consolidated Fund of India. IV. is required to withdraw money from the Contingency Fund of India. (a) I and III (b) I, II and III (c) I and II (d) I, II, III and IV 61. With reference to Indian Public Finance, consider the following statements: 1. Disbursements from Public Accounts of India are subject to the Vote of Parliament. 2. The Indian Constitution provides for the establishment of a Consolidated Fund, a Public Account and a Contingency Fund for each State. 3. Appropriations and disbursements under the Railway Budget are subject to the same form of parliamentary control as other appropriations and disbursements. Which of the statements given above are correct? (a) 1 and 2 (b) 2 and 3 (c) 1 and 3 (d) 1, 2 and 3
  • 6. Indian Polity For more info: www.iasnext.com Page 6 Ph. :0522 4241011/9454721860 62. Which of the following matters are not affected in case of dissolution of the Lok Sabha? 1. A bill originating and pending in the Rajya Sabha. 2. Pending notices, motions and resolutions in the Lok Sabha. 3. A joint sitting of Parliament if notification of such a sitting has been given before the dissolution. 4. Bills passed by both Houses and sent to the President for his assent. 5. Bills returned by President for reconsideration. (a) I, II and III (b) I, III, IV and V (c) II, III and IV (d) I, IV and V 63. Which one of the following statements is not correct? (a) In Lok Sabha, a no-confidence motion has to set out the grounds on which it is based (b) In the case of a no-confidence motion in Lok Sabha no conditions of admissibility have been laid down in the Rules (c) A motion of no-confidence, once admitted, has to be taken up within ten days of the leave being granted (d) Rajya Sabha is not empowered to entertain a motion of no-confidence 64. The two Houses of Parliament enjoy co-equal power in all spheres except: 1. financial matters 2. responsibility of the Council of Ministers 3. amendment procedure 4. election of President (a) III and IV (b) II, III and IV (c) I, II and III (d) I and II 65. What is the minimum age for being the member of the Parliament? (a) 21 years (b) 25 years (c) 30 years (d) 35 years 66. The total number of members in a Legislative Council should not exceed that of a Legislative Assembly by : (a) 1/3 (b) 2/3 (c) 1/2 (d) 1/4 67. Which one of the following is the largest (area wise) Lok Sabha constituency? (a) Kangra (b) Kachchh (c) Ladakh (d) Bhilwara 68. Match the following: Jurisdiction of the Supreme Court A. Original Jurisdiction B. Appellate Jurisdiction C. Advisory Jurisdiction Cases Covered 1. Advice on any question of law as may be referred to the Supreme Court for consideration by the President 2. Case involving interpretation of the Constitution 3. Appointment of officers and servants of the Supreme Court 4. Dispute between the Government of India and a State A B C (a) 1 2 3 (b) 4 3 2 (c) 4 2 1 (d) 2 1 3 69. Which of the following correctly reflects the position of the Upper House of the State Legislature as compared to the position of the Upper House of the Parliament? (a) 1/3rd members of Rajya Sabha retire every second year while 1/4th members of Legislative Council retire every 18 months (b) There is no provision for a joint sitting of the Legislative Assembly and the Legislative Council in the States for resolving deadlocks While there is such a provision in the case of the two House of Parliament (c) While no Bill other than a Money Bill can originate in the State Legislative Council, no Bill can originate in the Rajya Sabha (d) While Rajya Sabha has 12 nominated members, the State Legislative Council has none 70. If in an election to a State Legislative Assembly the candidate who is declared elected loses his deposit, it means that: (a) the polling was very poor (b) the election was for a multi- member constituency (c) the elected candidate's victory over his nearest rival was very marginal (d) a very large number of candidates contested the election 71. Match the following: A. Mandamus 1. Direction to an official for the performance of a duty B. Habeas Corpus 2. Release of an illegally detained person C. Certiorari 3. Transferring of a case from an inferior court to a court of higherjurisdiction D. Quo Warranto 4. Calling upon one to show by what authority he holds or claims afranchise or office A B C D (a) 1 2 3 4 (b) 3 2 1 4 (c) 4 1 3 2 (d) 4 3 2 1 72. Consider the following statements: 1. Justice V. R. Krishna Iyer was the Chief Justice of India 2. Justice V. R. Krishna Iyer is considered as one of the progenitors of public interest litigation(PIL) in the Indian judicial system. Which of the statements given above is/are correct? (a) 1 only (b) 2 only (c) Both 1 and 2 (d) Neither 1 nor 2
  • 7. Indian Polity For more info: www.iasnext.com Page 7 Ph. :0522 4241011/9454721860 73. A: The position of the legislative Council is inferior to that of the legislative Assembly. R1: The very existence of the Council depends on the will of the Assembly. R2: A Bill originating in the Council can be forthwith put to an end by the Assembly rejecting it. R3: One-sixth of the Council's members are nominated by the Governor. (a) A and R1, R2 and R3 are correct and R1, R2 and R3 explain A (b) A, R1 and R3 are correct and R1 and R3 explain A (c) A, R1, R2and R3are correct but only R1 and R2 explain A (d) A and R2 are wrong; R1 and R3 are correct 74. The High Courts in India were first started at: (a) Bombay, Madras, Calcutta (b) Delhi and Calcutta (c) Bombay, Delhi, Madras (d) Madras and Bombay 75. The number of seats in Vidhan Sabha is : (a) to be not more than five hundred and not less than 60 (b) to be not more than 500 and not less than 60 but an exception is recognished in the case of one State which has only 32 seats (c) to be not more than.600 and not less than 500 (d) varies from Vidhan Sabha to Vidhan Sabha 76. Who of the following shall cause every recommendation made by the Finance Commission to be laid before each House of Parliament? (a) The President of India (b) The Speaker of Lok Sabha (c) The Prime Minister of India (d) The Union Finance Minister 77. In which respect have the Centre-State relations been specifically termed as 'municipal relations'? (a) Centre's control of the State in the legislative sphere (b) Centre's control of the State in financial matters (c) Centre's control of the State in administrative sector (d) Centre's control of the State in planning process 78. Which of the following is/are extra constitutional and extra-legal device (s)for securing co-operation and co-ordination between the States in India? 1. The National Development Council. 2. The Governor's Conference. 3. Zonal Councils. 4. The Inter-State Council (a) I and II (b) I, III and IV (c) III and IV (d) Only IV 79. Match the following Year of Creation States A. 1960 1. Sikkim B. 1962 2. Goa C. 1975 3. Maharashtra D. 1987 4. Nagaland A B C D (a) 2 4 3 1 (b) 4 3 2 1 (c) 3 4 1 2 (d) 3 4 2 1 80. Match the following: A. Arunachal Pradesh 1. 22nd State B. Goa 2. 23rd State C. Mizoram 3. 24th State D. Sikkim 4. 25th State A B C D (a) 1 3 4 2 (b) 4 3 1 2 (c) 2 1 4 3 (d) 3 4 2 1 81. Corporation Tax: (a) is levied by the Union and collected and appropriated by the States (b) is levied by the Union and belongs to it exclusively (c) is levied and appropriated by the States (d) is levied by the Union and shared by the Union and the States 82. The States Reorganisation Act created ______ States and _______ Union Territories. (a) 14;7 (b) 14;6 (c) 15;6 (d) 15;7 83. Silvassa is the capital of (a) Lakshadweep (b) Tripura (c) Dadra and Nagar Haveli (d) Mizoram 84. What is the correct sequence of steps in electoral procedure? 1. Filing of nominations. 2. Presidential notification. 3. Scrutiny of nomination papers. 4. Withdrawal of candidature. 5. Allotment of symbols. (a) I, III, IV, V, II (b) II, I, IV, V, III (c) I, III, V, IV, II (d) II, I, III, IV, V 85. Match the following: A. Ex-Officio Chairman of Rajya Sabha 1. Speaker B. Presiding officer of the House of the People 2. Solicitor General C. First law officer of the State 3. Vice-President D. Representative of Government of India in important legal cases 4. Advocate General A B C D (a) 3 1 4 2 (b) 2 1 4 3 (c) 1 3 4 2
  • 8. Indian Polity For more info: www.iasnext.com Page 8 Ph. :0522 4241011/9454721860 (d) 4 1 3 2 86. What was the decision of the Supreme Court in Keshavanand Bharati case? (a) Parliament is supreme in the matters of legislation (b) In matters relating to compulsory acquisition of private property the court had the ultimate powers of determining what is public purpose under Article 31 (c) The power under Article 368 to amend the Constitution cannot be so exercised as to alter the basic structure or the essential features of the Constitution (d) The Supreme Court has full authority to pronounce on the Constitutional Validity of any State law 87. Which of the following are a source of income to the urban local bodies? 1. Octroi duty on goods brought into and taken out of the city. 2. Taxes on vehicles. 3. Income from water and electricity supply. 4. Financial grants from the State Governments. (a) I, II and III (b) II, III and IV (c) I, III and IV (d) I, II, III and IV 88. Match the following: A. 58th Amendment 1. Delhi to be called as National Capital Territory of Delhi B. 61st Amendment 2. Reduced voting age from 21 years to 18 years C. 69th Amendment 3. An authoritative text of the Constitution in Hindi D. 71st Amendment 4. Included Konkani, Manipuri and Nepali languages in the Eighth Schedule A B C D (a) 3 2 1 4 (b) 3 1 2 4 (c) 2 1 3 4 (d) 4 2 1 3 89. Which of the two words among the following were added to the Preamble to the Constitution of India by the Constitution (42nd Amendment) Act, 1976? (a) Sovereign and Socialist (b) Socialist and Democratic (c) Socialist and Secular (d) Secular and Democratic 90. The President of India appoints the Chief Election Commissioner, but he cannot be removed from office except by a special procedure laid down in the Constitution. This procedure is the same as the one prescribed for the removal of the (a) Vice-President (b) Judges of the Supreme Court (c) Members of the State Public Service Commissions (d) Members and Chairman of the UPSC 91. The theory of "basic structure" of the Constitution was propounded by the Supreme Court in the (a) A.K.Gopalan Case (b) Golaknath Case (c) Kesavananda Bharati Case (d) Minerva Mills Case 92. The Comptroller and Auditor General (CAG) does not audit the receipts and expenditure of (a) Municipal Undertakings (b) State Governments (c) Government Companies (d) Union Government 93. The Constituent Assembly of India was converted into the Provisional Parliament of India on (a) 1.1.1950 (b) 15.8.1947 (c) 2.10.1950 (d) 26.1.1950 94 .The Union or India consists of _____ States and Union Territories. (a) 22 : 8 (b) 24 : 7 (c) 28 : 7 (d) 21 : 8 95. During the period of 15th August 1947 to the 26th January 1950, the political status of India was that of a (a) Sovereign Republic (b) Dominion in the British Commonwealth of Nations (c) Sovereign Republic and a member of the Commonwealth (d) Sovereign State 96. Consider the following tasks: 1) Superintendence, direction and conduct of free and fair elections. 2) Preparation of electoral rolls for all elections to the Parliament, State-Legislatures and the Office of the President and the Vice-President. 3) Giving recognition to political parties, allotting election symbols to political parties and individuals contesting the election. 4) Proclamation of final verdict in the case of election disputes. Which of the above are the functions of the Election Commission of India? (a) 1, 2 and 3 (b) 2, 3 and 4 (c) 1 and 3 (d) 1, 2 and 4 97. In the Government of India, under which Ministry is the National River Conservation Directorate? (a) Ministry of Agriculture (b) Ministry of Earth Sciences (c) Ministry of Water Resources (d) Ministry of Environment and Forests
  • 9. Indian Polity For more info: www.iasnext.com Page 9 Ph. :0522 4241011/9454721860 98. The seven Union Territories occupy roughly ______ per cent of the total area of the country. (a) 12 (b) 0.33 (c) 6 (d) 3 Ans. b 99. The first Independence Day (August 15, 1947) fell on a (a) Wednesday (b) Thursday (c) Friday (d) Saturday 100. Match the following: Prominent Functionaries Term of Office A. Supreme Court Judge 1. Till 65 years of age B. High Court Judge 2. Till 62 years of age C. Comptroller and 3. 6 years or age of 65 Auditor General of India whichever is less D. Governor 4. 5 years A B C D (a) 1 2 3 4 (b) 1 3 2 4 (c) 2 1 3 4 (d) 3 2 4 1 101. Match the following: A. Madhu Limayi 1. A former Vice-President B. Pattabhi Sitaramaiah 2. The first Speaker of the Lok Sabha C. B.D. Jatti 3. A veteran parliamentarian of yester years D. Mavlankar 4. Author of a famous book on the history of Indian National Congress A B C D (a) 2 4 3 1 (b) 3 1 2 4 (c) 4 3 1 2 (d) 3 4 1 2 102. In India, if a religious sect/community is given the status of a national minority, what special advantages is it entitled to ? 1. It can establish and administer exclusive educational institutions. 2. The President of India automatically nominates a representative of the community to Lok Sabha. 3. It can derive benefits from the Prime Minister's 15-Point Programme. Which of the statements given above is/are correct? (a) 1 only (b) 2 and 3 only (c) 1 and 3 only (d) 1, 2 and 3 103. Which of the following condition/conditions must be fulfilled by the NRIs to be eligible to vote in elections in India? 1. They must be physically present in their place of origin to exercise their franchise. 2. NRIs whether they have acquired citizenship of other countries or not are eligible to vote 3. Eligible NRIs have to register by filling up form 6-A with electoral registration office. Select the correct answer using the code given below: (a) 1, 2 and 3 (b) 1 and 3 only (c) 2 only (d) 3 only 104. Which one of the following States has granted Sanskrit language the status of the second official language of the State? (a) Bihar (b) Chhattisgarh (c) Uttar Pradesh (d) Uttarakhand 105. Which of the following Constitution Amendment Acts seeks that the size of the Councils of Ministers at the Centre and in a State must not exceed 15 percent of the total number of members in the Lok Sabha and the total number of members of the Legislative Assembly of that State, respectively? (a) 91st (b) 93rd (c) 95th (d) 97th 106. Which one of the following is the correct sequence in the descending order of precedence in the warrant of precedence? (a) Attorney General of India - Judges of the Supreme Court - Members of Parliament Deputy Chairman of Rajya Sabha (b) Judges of the Supreme Court - Deputy Chairman of Rajya Sabha - Attorney General of India - Members of Parliament (c) Attorney General of' India - Deputy Chairman of Rajya Sabha - Judges of the Supreme Court - Members of Parliament (d) Judges of the Supreme Court - Attorney General of India - Deputy Chairman of Rajya Sabha - Members of Parliament 107. The Constitution (Seventy-Third Amendment) Act, 1992, which aims at promoting the Panchayati Raj Institutions in the country, provides for which of the following? 1. Constitution of District Planning Committees. 2. State Election Commissions to conduct all panchayat elections. 3. Establishment of State Finance Commissions. Select the correct answer using the codes given below: (a) 1 only (b) 1 and 2 only (c) 2 and 3 only (d) 1, 2 and 3 108. For which one of the following reforms was a Commission set up under the Chairmanship of Veerappa Moily by the Government of India? (a) Police Reforms (b) Tax Reforms (c) Reforms in Technical Education (d) Administrative Reforms 109. Which of the following is incorrect in respect of Local Government in India? (a) According to the Indian Constitution local government is not an independent tier in the federal system (b) 30% of the seats in local bodies are reserved for women (c) Local government finances are to be provided by a Commission (d) Elections to local bodies are to be determined by a Commission
  • 10. Indian Polity For more info: www.iasnext.com Page 10 Ph. :0522 4241011/9454721860 110. The expression 'Creamy layer' used in the judgement of the Supreme Court relating to the case regarding reservations refers to: (a) those sections of the society which pay income tax (b) those sections of socially and educationally backward classes of the society that are developed (c) those sections of the society that are considered advanced according to the Karpuri Thakur formula (d) all sections of the upper castes of the society 111. The composition of the UPSC is: (a) laid down in the Constitution (b) determined by Parliament (c) determined by the President (d) determined by the Chairman of the UPSC 112. Which of the following is correct regarding booth capturing? 1. It has been defined in the Constitution after the 61st amendment. 2. It includes the seizure of a polling booth to prevent the orderly conduct of elections. 3. It is also committed when any elector is threatened and prevented from going to the polling station to cast his vote. 4. It has been declared a cognizable offence punishable by imprisonment. (a) 2, 3, and 4 (b) 1, 2 and 3 (c) 2 and 3 (d) 1, 2, 3 and 4 113. The first Law Officer of the Government of India is: (a) Chief Justice of India (b) Law Minister (c) Attorney General (d) Auditor General 114. The declaration which outlines the future programme and policy of a political party issued on the eve of a general election is called: (a) white paper (b) manifesto (c) yellow paper (d) mandate 115. The old name of which State/Union Territory is wrongly given? (a) Karnataka - Mysore (b) Tamil Nadu - Madras (c) Lakshadweep - Laccadive, Minicoy and Amindiv Islands (d) Meghalaya - Eastern Hill Province 116. Who of the following shall cause every recommendation made by the Finance Commission to be laid before each House of Parliament? (a) The President of India (b) The Speaker of Lok Sabha (c) The Prime Minister of India (d) The Union Finance Minister 117. Which of the following features is/are contrary to the norms of a federal polity? 1. Common All India Service 2. Single integrated judiciary Select the correct answer using the code given below: (a) 1 only (b) 2 only (c) Both 1 and 2 (d) Neither 1 nor 2 118. Which one of the following was an associate State of India before becoming a full fledged State? (a) Meghalaya (b) Mizoram (c) Manipur (d) Sikkim 119. The distribution of power between Centre and the States is based on the scheme provided in : (a) Government of India Act, 1935 (b) Montague-Chelmsford Act, 1919 (c) Minto-Morley Reforms, 1909 (d) Indian Independence Act, 1947 120. The provisions regarding division of taxes between Union and the States: (a) can be suspended during National Emergency (b) can be suspended during Financial Emergency (c) can be suspended only with the consent of the majority of State legislatures (d) cannot be suspended under' any circumstances